Spine Flashcards

1
Q

A 45-year-old woman undergoes an uncomplicated microdiscectomy of the lumbar spine in prone
position. Postoperatively, she awakens with severe, right-sided pain in the neck, parascapular
region, and arm. Neurological examination shows weakness of shoulder abduction and sensory
loss along the medial forearm and small finger. Which of the following is the most likely diagnosis?
Answers:
A. Lumbar spinal fluid leakage
B. Intracranial hemorrhage
C. Postoperative hematoma
D. Cervical disc herniation
E. Brachial plexus stretch injury

A

Brachial plexus stretch injury

Discussion:
Surgery in prone position with arms abducted to 90 degrees, such as is routinely performed for
lumbar microdiscectomy, is associated with the highest risk of brachial plexus stretch injury.
The brachial plexus can be compressed between the clavicle and first rib when abducting the arm
more than 90 degrees, and pressure points at the clavicle can exacerbate this; both should be
routinely avoided in spine surgery positioning. Management is supportive with physical therapy to
retain range of movement and pain control. Prognosis for recovery is good but can take months.
Sensory recovery usually occurs first followed by motor function return in lower cervical roots first
followed by upper cervical roots.
The distribution of neurologic deficits in both the C5 myotome (shoulder abduction) and C8
dermatome is not consistent with isolated cervical disc herniation. Lumbar spinal CSF leakage or
postoperative hematoma would not be expected to cause unilateral upper extremity neurologic
deficits. Intracranial hemorrhage is highly unlikely to cause isolated deficits as described in this
patient.
References:
Uribe JS, Kolla J, Omar H, Dakwar E, Abel N, Mangar D, Camporesi E. Brachial plexus injury
following spinal surgery. J Neurosurg Spine. 2010 Oct;13(4):552-8. doi:
10.3171/2010.4.SPINE09682. PMID: 20887154.
Pubmed Web link
https://pubmed.ncbi.nlm.nih.gov/20887154/
Chung I, Glow JA, Dimopoulos V, Walid MS, Smisson HF, Johnston KW, Robinson JS, Grigorian
AA. Upper-limb somatosensory evoked potential monitoring in lumbosacral spine surgery: a
prognostic marker for position-related ulnar nerve injury. Spine J. 2009 Apr;9(4):287-95. doi:
10.1016/j.spinee.2008.05.004. Epub 2008 Aug 5. PMID: 18684675.
Pubmed Web link
https://pubmed.ncbi.nlm.nih.gov/18684675/

How well did you know this?
1
Not at all
2
3
4
5
Perfectly
2
Q

A 45-year-old man with a known Meyerding Grade 1 L5-S1 spondylolisthesis with associated L5-
S1 spinal stenosis has a six-month history of axial low back pain and bilateral radicular pain
radiating to the buttocks and posterior thighs. During the past three months, treatment with
pharmacotherapy, physical therapy, and epidural corticosteroid injections has not caused
significant relief. Which of the following is the most appropriate next step in management?
Answers:
A. L4-S1 decompression and fusion with pelvic supplemental fixation
B. Vertebroplasty
C. Radiofrequency ablations
D. L5-S1 decompression and fusion
E. Chronic opioid therapy

A

L5-S1 decompression and fusion

Discussion:
The patient has a grade 1 L5-S1 spondylolisthesis along with spinal stenosis at the segment. He
has mechanical back pain and neurogenic claudication, and he has failed 3 months of
nonoperative treatment with physical therapy, epidural steroid injections, and pharmacotherapy.
Therefore, surgical treatment would be recommended, which is typically a 1-level decompression
and fusion, often with an interbody fusion, although this can be avoided if there are not pars
defects. Although some patients with low grade spondylolisthesis can have good outcomes
following decompression alone, a recent randomized trial showed improved outcomes with
decompression and fusion. A more extensive construct with pelvic supplemental fixation is
generally indicated for a high-grade spondylolisthesis but would not be indicated in this case. The
patient has already failed 3 months of nonoperative treatments, so additional nonoperative
treatment or long-term opioid therapy would be a poor choice.
References:
Ghogawala Z, Dziura J, Butler WE, Dai F, Terrin N, Magge SN, Coumans JV, Harrington JF, AminHanjani S, Schwartz JS, Sonntag VKH, Barker FG, Benzel EC. Laminectomy plus Fusion versus
Laminectomy Alone for Lumbar Spondylolisthesis. N Engl J Med. 2016 Apr 14;374(15)1424-34.
Pubmed Web link
https://pubmed.ncbi.nlm.nih.gov/27074067/
Forsth P, Olafsson G, Carlsson T, Frost A, Borgstrom F, Fritzell P, Ohagen P, Michaelsson K,
Sanden B. A Randomized, Controlled Trial of Fusion Surgery for Lumbar Spinal Stenosis. N ENgl
J Med. 2016 Apr 14;374(15):1413-23.
Pubmed Web link
https://pubmed.ncbi.nlm.nih.gov/27074066/

How well did you know this?
1
Not at all
2
3
4
5
Perfectly
3
Q

Examination of which of the following movements is most helpful in distinguishing an L4
radiculopathy from a proximal femoral neuropathy?
Answers:
A. Thigh adduction
B. Hip flexion
C. Knee extension
D. Ankle plantar flexion
E. Knee flexion

A

Hip flexion

Discussion:
A femoral neuropathy will tend to cause weakness in hip flexion (iliopsoas muscle) and knee
extension (quadriceps muscle). Hip flexion is mediated by the L1 and L2 nerve roots, whereas
knee extension is mediated by the L3 and L4 nerve roots. Thus, an L4 radiculopathy will not
generally affect hip flexion, but will affect knee extension. A femoral neuropathy on the other hand
will affect both. An exception to this occurs when a femoral neuropathy occurs at the level of the
inguinal ligament. In this circumstance, hip flexion will be spared, and thus will mimic an L4
radiculopathy more closely. In these cases, EMG sampling of the lumbar paraspinal muscles can
be helpful. If denervation changes are present in paraspinal muscles, that suggests a nerve root
level problem rather than a peripheral nerve problem. Thigh adduction is supplied by the L2-L4
nerve roots via the obturator, sciatic, and femoral nerves, so weakness of that movement is fairly
nonspecific. Knee flexion is mediated by the L5-S2 nerve roots via the sciatic nerve, and thus
would not be helpful here. Ankle plantar flexion is mediated by the S1-S2 nerve roots via the
sciatic nerve, and thus would also not be helpful here.
References:
nul

How well did you know this?
1
Not at all
2
3
4
5
Perfectly
4
Q

A 45-year-old man is seen in the post-anesthesia care unit after undergoing an anterior cervical
discectomy and fusion with plating at C6-7. Physical examination shows constricted pupil and
ptosis. Which of the following is the most likely explanation of this patient’s condition?
Answers:
A. Vertebral artery dissection
B. Embolization of carotid plaque
C. Facial nerve injury
D. Sympathetic plexus injury
E. Vagus nerve injury

A

Sympathetic plexus injury

Discussion:
Postoperative Horner’s syndrome following ACDF is a rare complication that occurs in less than
0.1% of patients. The sympathetic plexus or trunk is located running on the longus colli muscle. In
anterior cervical surgery, the sympathetic plexus is at risk of injury due to unintentional thermal
injury from using monopolar electrocautery directly on the longus colli muscle, or from prolonged
retraction inducing stretching of the fibers. Most cases of Horner’s syndrome improve or resolve
spontaneously within 3 to 6 months following the injury. Facial nerve injury is unlikely with a midcervical dissection. Vagus nerve injury (most commonly the recurrent laryngeal nerve), results in
hoarse voice, dysautonomia and other systemic symptoms. Horner’s syndrome is unlikely to be
caused by an embolism from either carotid or vertebral arteries.
References:
Weinberg DS, Morris WZ, Gebhart JJ, Liu RW. Pelvic incidence: an anatomic investigation of 880
cadaveric specimens. Eur Spine J. 2016 Nov;25(11):3589-3595. Epub 2015 Nov 4. Inami S,
Moridaira H, Takeuchi D, et al. Postoperative Status of Global Sagittal Alignment with
Compensation in Adult Spinal Deformity. Spine (Phila Pa 1976). 2018 Apr 16.

How well did you know this?
1
Not at all
2
3
4
5
Perfectly
5
Q

During a surgical approach for an anterior lumbar interbody fusion at L4-5, which of the following
blood vessels must be identified and ligated to avoid injury?
Answers:
A. Distal aorta
B. Genitofemoral nerve
C. Hypogastric plexus
D. Iliolumbar vein
E. Vena cava

A

Iliolumbar vein

Discussion:
The autonomic nerves (i.e. hypogastric plexus) traverse the prevertebral space at L5-S1 and may
be susceptible to thermal injury (e.g. monopolar cautery) resulting in retrograde ejaculation in men.
At L4-5, the left common iliac vessels traverse the prevertebral space. The vena cava has
bifurcated rostral to this level. The left iliolumbar vein enters the left common iliac vein laterally, and
often must be ligated before mobilizing the iliac vessels. At L3-4, a minimal amount of mobilization
of the distal aorta from left to right is necessary. The genitofemoral nerve is not routinely visualized
during exposure of L5-S1 for anterior lumbar interbody fusion. It is an important structure at risk
during lateral lumbar interbody fusion exposure.
References:
Sasso RC, Kenneth Burkus J, LeHuec JC. Retrograde ejaculation after anterior lumbar interbody
fusion: transperitoneal versus retroperitoneal exposure. Spine (Phila Pa 1976). 2003 May
15;28(10):1023-6.
Fantini GA, Pappou IP, Girardi FP, Sandhu HS, Cammisa FP Jr. Major vascular injury during
anterior lumbar spinal surgery: incidence, risk factors, and management. Spine (Phila Pa 1976).
2007 Nov 15;32(24):2751-8.

How well did you know this?
1
Not at all
2
3
4
5
Perfectly
6
Q

A 65-year-old woman with type 2 diabetes mellitus is evaluated because of a three-week history of
worsening back pain with radiation to the legs, urinary retention, perineal numbness, and
diminished rectal tone. Erythrocyte sedimentation rate, C-reactive protein, and white blood cell
count are increased. An MR image of the lumbosacral spine is shown. Which of the following is the
most appropriate next step in management?
Answers:
A. Antibiotic Therapy
B. Interventional Radiology Guided Aspiration/Biopsy
C. Conservative management with physical therapy 
D. Anterior Lumbar discectomy and fusion
E. Laminectomy and Surgical Debridement

A

Laminectomy and Surgical Debridement

Discussion:
The patient presents with acute neurological deficits consistent with compression of the cauda
equina. MRI imaging reveals a posterior lumbar epidural compressive lesion, concerning for
epidural abscess in the setting of an elevated white blood cell count and inflammatory markers.
Given acute neurological deficits, the next step in management would be urgent surgical
decompression of the thecal sac and incision and drainage of the epidural abscess. Since the
lesion is located in the posterior epidural space, a posterior approach would be recommended.
Antibiotic therapy would be initiated in the post-operative period and continued for 6-12 weeks post
operatively depending on organism and response to treatment; however, given the acute
neurological compromise, surgery would be recommended first for washout and decompression.
An interventional radiology guided aspiration/biopsy would not be recommended given the acute
decompensation in neurological function/exam and that a specimen can be obtained for culture at
the time of surgery.
References:
Darouiche RO. Spinal epidural abscess. N Engl J Med. 2006 Nov 9;355(19):2012-20. doi:
10.1056/NEJMra055111. PMID: 17093252.
Pubmed Web link
https://pubmed.ncbi.nlm.nih.gov/17093252/
Babic M, Simpfendorfer CS, Berbari EF. Update on spinal epidural abscess. Curr Opin Infect Dis.
2019 Jun;32(3):265-271. doi: 10.1097/QCO.0000000000000544. PMID: 31021957
Pubmed Web link
https://pubmed.ncbi.nlm.nih.gov/31021957/

How well did you know this?
1
Not at all
2
3
4
5
Perfectly
7
Q

A patient presents following a thoracic stab wound with right lower extremity weakness. In the
trauma bay, decreased pain sensation on the left side below the wound is noted. The clinical
syndrome that best describes this constellation of symptoms is
Answers:
A. Cauda equina syndrome
B. Brown-Sequard syndrome
C. Anterior cord syndrome
D. Conus medullaris syndrome
E. Central cord syndrome

A

Brown-Sequard syndrome

Discussion:
This is a classic example of a Brown-Sequard syndrome, also known as hemi-cord syndrome.
Brown-Sequard syndrome is a neurological condition characterized by injury to one half of the
spinal cord, resulting in ipsilateral weakness or paralysis (due to interruption of the lateral
corticospinal tracts), ipsilateral loss of tactile discrimination and vibratory sense (interruption of
posterior white columns), and contralateral loss of pain and temperature (interruption of lateral
spinothalamic tracts). Brown-Sequard syndrome may be caused by a spinal cord tumor, trauma
(such as a puncture wound to the neck or back), ischemia (obstruction of a blood vessel), infection
such as tuberculosis, or inflammation such as multiple sclerosis. Central cord injury occurs in the
setting of hyperextension of the neck in an already stenotic cervical spine. The classic findings are
weak distal upper extremities and/or weak proximal lower extremities. The proximal upper
extremities and distal lower extremities are usually unaffected; however, when they are involved,
these muscle groups are stronger than the distal upper extremity muscle groups and proximal
lower extremity muscle groups respectively. A central cervical herniated disk will cause upper
motor neuron signs with clumsiness of hands and or feet, but will not give the classic clinical
findings of central cord syndrome. Anterior cord syndrome is a vascular injury associated with loss
of motor function with retained sensory function (consistent with preservation of dorsal vascular
supply to the dorsal columns). Cauda equina syndrome is a typically a surgical emergency. In
patients with cauda equina syndrome, a lesion compresses the caudal spinal nerve roots (disc
most commonly), and can present with severe pain, weakness and loss of bowel or bladder
function. Surgical decompression is required to prevent permanent dysfunction. Conus medullaris
syndrome can present with symptoms similar to cauda equina syndrome but with inclusion of
upper motor neuron findings.
References:
nul

How well did you know this?
1
Not at all
2
3
4
5
Perfectly
8
Q

A 61-year-old woman with terminal bronchial carcinoma is evaluated for unrelenting right chest
and shoulder pain. She has multiple metastatic lesions in the ribs, humerus, and scapula. Medical
management of her constant pain is ineffective. Which of the following is the correct spinal cord
location for percutaneous cervical cordotomy?
Answers:
A. Ipsilateral spinothalamic tract
B. Dorsal root entry zone
C. Ipsilateral corticospinal tract
D. Contralateral spinothalamic tract
E. Dorsal column

A

Contralateral spinothalamic tract

Discussion:
Percutaneous cordotomy targets the contralateral spinothalamic tract in patients with unilateral
somatic pain, primarily cancer related pain. First order spinal nerve axons enter the ipsilateral
dorsal root, before they terminate in the dorsal horn. Second order axons then travel across the
anterior white commissure and ascend via the spinothalamic tract somatotopically. Percutaneous
cordotomy is typically performed using CT guidance with needle entry at the C1-2 level.
References:
Ryken TC, Aarabi B, Dhall SS, Gelb DE, Hurlbert RJ, Rozzelle CJ, Theodore N, Walters BC,
Hadley MN. Management of isolated fractures of the atlas in adults. Neurosurgery. 2013 Mar;72
Suppl 2:127-31. doi: 10.1227/NEU.0b013e318276ee2a. PMID: 23417185.
Kakarla UK, Chang SW, Theodore N, Sonntag VK. Atlas fractures. Neurosurgery. 2010 Mar;66(3
Suppl):60-7. doi: 10.1227/01.NEU.0000366108.02499.8F. PMID: 20173529

How well did you know this?
1
Not at all
2
3
4
5
Perfectly
9
Q

Which of the following cervical vertebrae is the most common site for the entry of the vertebral
artery into the vertebral foramen?
Answers:
A. C5
B. C6
C. C7
D. C4
E. C3

A

C6

Discussion:
The vertebral artery enters the C6 transverse foramen in approximately 93% of the population.
Anatomic variants, including entry at C7 (5% of individuals), C5, and C4 can also be seen.
References:
Argenson G, Franche P, Sylla S, et al: The vertebral arteries (segments V1 and V2). Anatomical
Clinical 1980; 2:29-41. Uchino A, Saito N, Takahashi M, Okada Y, Kozawa E, Nishi N, Mizukoshi
W, Nakajima R, Watanabe Y. Variations in the origin of the vertebral artery and its level of entry into
the transverse foramen diagnosed by CT angiography. Neuroradiology. 2013 May;55(5):585-94.
doi: 10.1007/s00234-013-1142-0. Epub 2013 Jan 24. PMID: 23344682.
Copy

How well did you know this?
1
Not at all
2
3
4
5
Perfectly
10
Q

The left anterolateral spinothalamic tract at the T6 level transmits which of the following types of
information?
Answers:
A. Light touch
B. Pain
C. Proprioception
D. Stereognosis
E. Vibration

A

Pain

Discussion:
The lateral spinothalamic tract carries information about pain and temperature. The anterior
spinothalamic tract carries sensory information regarding crude touch. This information is carried in
slow-conducting fibres (Aδ and C fibres) in contrast to the rapidly conducting fibres carrying
information about pain and temperature. After joining the spinal cord, the fibres cross after
ascending 1–2 segments and synapse in Lissauer’s tract. From there, the fibres ascend as the
lateral or anterior spinothalamic tract, and terminate in the ventral posterior nucleus of the
thalamus. Fibres are also given off to the reticular formation and periaqueductal grey matter. The
sensory cerebral cortex receives the final projections as described above. The gracile and cuneate
tracts carry information about proprioception and light touch and stereognosis.
References:
Gulgun Kayalioglu, Chapter 10 - Projections from the Spinal Cord to the Brain, Editor(s): Charles
Watson, George Paxinos, Gulgun Kayalioglu, The Spinal Cord, Academic Press, 2009, Pages
148-167. Stewart Hendry, Steven Hsiao,
Chapter 24 - The Somatosensory System,
Editor(s): Larry R. Squire, Darwin Berg, Floyd E. Bloom, Sascha du Lac, Anirvan Ghosh, Nicholas
C. Spitzer,
Fundamental Neuroscience (Fourth Edition),
Academic Press,
2013,
Pages 531-551,
ISBN 9780123858702. Lisa Harvey,
Chapter 1 - Background information,
Editor(s): Lisa Harvey,
Management of Spinal Cord Injuries,
Churchill Livingstone,
2008,
Pages 3-33,
ISBN 9780443068584,
https://doi.org/10.1016/B978-0-443-06858-4.50007-1
https://doi.org/10.1016/B978-0-12-385870-2.00024-X.

How well did you know this?
1
Not at all
2
3
4
5
Perfectly
11
Q

Thoracic pedicle morphology studies consistently show that the narrowest pedicle is most likely
to be found at which of the following locations?
Answers:
A. T12
B. T10
C. T1
D. T8
E. T5

A

T5

Discussion:
The correct answer is T5 as the transverse diameter of the thoracic pedicle decreases from T1 to
its narrowest diameter between T4-6. From T7 -T12 the diameter of the thoracic pedicles
progressively enlarge. This has implications for pedicle screw placement, in which narrow pedicle
anatomy in the mid-thoracic spine may preclude standard transpedicular screw insertion
techniques or require narrower diameter screws.
References:
McLain RF, Ferrara L, Kabins M. Pedicle morphometry in the upper thoracic spine. Spine. 2002
Nov 15;27(22):2467-71. McCormack, Bruce, M., et al. “Anatomy of the Thoracic Pedicle”.
Neurosurgery, vol. 37, no. 2, August 1995, pp. 303–308

How well did you know this?
1
Not at all
2
3
4
5
Perfectly
12
Q

Among the following primary tumors, which are considered to be radiosensitive to the
administration of conventional external beam radiation therapy in the context of spinal metastatic
disease
Answers:
A. Multiple myeloma
B. Melanoma
C. Renal cell carcinoma
D. Sarcoma
E. Non-small cell lung carcinoma

A

Multiple myeloma

Discussion:
The advent of SRS has resulted in durable long-term control of spinal metastatic disease even
among patients who harbor metastatic disease from classically radio-resistant primary tumors.
Despite advances in radiosurgery, patients with radiosensitive tumors can be treated with cEBRT
regardless of the degree of ESCC. Among the presented choices, multiple myeloma is classically
considered to be a radiosensitive pathology, as are lymphoma and plasmacytoma.
References:
nul

How well did you know this?
1
Not at all
2
3
4
5
Perfectly
13
Q

Which of the following interbody graft materials has the greatest compressive strength?
Answers:
A. Carbon fiber-reinforced PEEK
B. Cancellous bone
C. Titanium
D. Poly-ether-ether-ketone (PEEK)
E. Cortical bone

A

Titanium

Discussion:
Out of the common interbody graft materials, titanium has the highest compressive strength, with
an elastic modulus >50 GPa. Cancellous allograft bone has the lowest elastic modulus, around 4
GPa, similar to PEEK. Carbon fiber reinforced PEEK and cortical bone have similar elastic moduli,
in the 15-20 GPa range.
References:
Heary RF, Parvathreddy N, Sampth S, Agarwal N. Elastic modulus in the selection of interbody
implants. J Spine Surg. 2017 Jun; 3(2):163-167
Pubmed Web link
https://www.ncbi.nlm.nih.gov/pmc/articles/PMC5506312/
Niinomi M, Liu Y, Nakai M, Liu H, Li H. Biomedical titanium alloys with Young’s moduli close to that
of cortical bone. Regen Biomater. 2016 Sep;3(3):173-85.
Pubmed Web link
https://pubmed.ncbi.nlm.nih.gov/27252887/

How well did you know this?
1
Not at all
2
3
4
5
Perfectly
14
Q

When nociceptors are activated, which of the following is the primary neurotransmitter facilitating
synaptic transmission to dorsal horn spinal cord neurons?
Answers:
A. Dopamine
B. Norepinephrine
C. GABA
D. Serotonin
E. Glutamate

A

Glutamate

Discussion:
Nociceptors are excitatory neurons that release glutamate as their primary neurotransmitter in
addition to various peptides such as somatostatin and substance P. They are found in a variety of
body tissues, and relay nociceptive information (pain). They are subdivided by the type of
nociceptive information they relay, such as high threshold mechanoreceptors, thermal receptors,
chemical receptors, polymodal receptors, and silent receptors.
References:
Brain WR, Northfield D, Wilkinson M. The neurological manifestations of cervical spondylosis.
Brain 75: 187-225, 1952

How well did you know this?
1
Not at all
2
3
4
5
Perfectly
15
Q

Following an uneventful bilateral high cervical cordotomy, a 35-year-old patient with metastatic
non-squamous cell lung carcinoma dies unexpectedly during the night. Which of the following is
the most likely cause of death?
Answers:
A. Spinocerebellar tract lesion
B. Primary pulmonary disease
C. Lesion extending to the reticulospinal tract
D. Dorsolateral tract of Lissauer interruption
E. Sympathetic chain disruption

A

Lesion extending to the reticulospinal tract

Discussion:
Cordotomy is mechanical disruption of the nociceptive fibers in the spinothalamic tract for relief of
intractable pain, most often due to malignancy. Deafferentation pain and visceral pain are not
responsive to cordotomy because these sensations are not conveyed in the spinothalamic tract.
The indication for cordotomy is severe unremitting pain that is not controlled by oral/intravenous
/intrathecal analgesics, peripheral nerve anesthetic blocks, or implantable dorsal column
stimulator.
Cordotomy can be performed via posterior open, percutaneous, or endoscopic approach, or via
transdiscal approach. The goal is mechanical disruption of the spinothalamic and spinoreticular
nociception pathways in the anterolateral column to diminish pain sensation, most often via imageguidance with a needle. Because of the organization of the Lissauer tract, cordotomy results in
decreased pain and temperature sensation at levels three or four segments below the level of
cordotomy on the contralateral side.
In the upper cervical spinal cord, respiratory fibers of the reticulospinal tract are located just medial
to the spinothalamic tract. If the lesion extends to the reticulospinal tract, this can lead to
respiratory failure and death.
References:
Reference (1)
Javed S, Viswanathan A, Abdi S. Cordotomy for Intractable Cancer Pain: A Narrative Review. Pain
Physician. 2020 Jun;23(3):283-292. PMID: 32517394.
Pubmed Web link
https://pubmed.ncbi.nlm.nih.gov/32517394/
Reference (2)
Lahuerta J, Buxton P, Lipton S, Bowsher D. The location and function of respiratory fibres in the
second cervical spinal cord segment: respiratory dysfunction syndrome after cervical cordotomy. J
Neurol Neurosurg Psychiatry. 1992 Dec;55(12):1142-5. doi: 10.1136/jnnp.55.12.1142. PMID:
1479392; PMCID: PMC1015328.
Pubmed Web link
https://pubmed.ncbi.nlm.nih.gov/1479392

How well did you know this?
1
Not at all
2
3
4
5
Perfectly
16
Q

In a patient with a lumbar flat back, which of the following techniques will provide the greatest
degree of correction of the sagittal vertical axis?
Answers:
A. Multi-level laminectomy
B. Vertebral column resection
C. Pedicle subtraction osteotomy
D. Laminectomy
E. Smith Peterson osteotomy

A

Vertebral column resection

Discussion:
Osteotomies for sagittal deformity correction can be classified using the Schwab osteotomy
classification, which is based on the degree of bony and or disc resection. Smith Peterson
osteotomy offers 10-15 degrees of sagittal correction. Pedicle subtration osteotomy allows up to
30-35 degrees correction. Vertebral column resection allows 40+ degrees of correction.
Laminectomies alone offer no significant correction.
References:
Ditunno JF Jr, Young W, Donovan WH, et al. The international standards booklet for neurological
and functional classification of spinal cord injury. American Spinal Injury Association. Paraplegia.
1994 Feb;32(2):70-80. Russell, S., 2016. Examination of peripheral nerve injuries.

How well did you know this?
1
Not at all
2
3
4
5
Perfectly
17
Q

Which of the following spinopelvic parameters remains constant before and after spinal deformity
surgery?
Answers:
A. Pelvic Tilt
B. Sacral Slope
C. Lumbar Lordosis
D. Pelvic Incidence
E. Thoracic kyphosis

A

Pelvic Incidence

Discussion:
Pelvic incidence stays constant and is a fixed measurement of the sacral slope and pelvic tilt. The
incidence is always the same pre- and post-operatively. Pelvic tilt, lumbar lordosis, sacral slope
and sagittal balance can all change during and after deformity spine surgery as correction is
achieved.
References:
null

How well did you know this?
1
Not at all
2
3
4
5
Perfectly
18
Q

A 60-year-old man who has multiple sclerosis and an indwelling baclofen pump undergoes surgery
for lumbar neurogenic claudication. The evening following surgery, the nurse requests a sedative
for the patient. She reports that he is tachycardic and febrile; hallucinations and increasing
agitation are noted. On evaluation, he is found to be protecting his airway and has full strength, but
increased spasticity. After transfer to the ICU, which of the following is the most appropriate next
step in management?
Answers:
A. Return to surgery for suspected infection
B. Empiric antibiotic therapy
C. High-dose intravenous benzodiazepines
D. Diagnostic lumbar puncture
E. Observation

A

High-dose intravenous benzodiazepines

Discussion:
A patient with indwelling baclofen intrathecal catheter and lumbar spine surgery has high risk for
unintentional catheter dislodgement or disruption even if not directly visualized in the surgical field,
due to retraction or tension. Intrathecal catheters typically are placed to enter the thecal sac in the
lumbar spine to avoid the risk of spinal cord injury, and then the tubing is advanced cranially in the
subarachnoid space to the desired catheter tip location. The onset of severe symptoms within the
day of surgery is more rapid than what one would expect for surgical site infection or meningitis.
Intrathecal baclofen withdrawal can lead to multi-system organ failure and be fatal if unrecognized.
An immediate step in management of suspected baclofen withdrawal is institution of high-dose oral
baclofen and high-dose intravenous benzodiazepines. Very often, even high doses of oral baclofen
will not be effective in managing withdrawal, because patients with chronic intrathecal baclofen
administration have down-regulation of central GABAB receptors and the CNS bioavailability of
oral baclofen is significantly lower than intrathecal baclofen.
Benzodiazepines are helpful in controlling spasticity and seizures during acute management of
suspected intrathecal baclofen withdrawal syndrome via action on GABAA receptors of the spinal
cord as well as central receptors. To reduce risk of seizures and spasticity, benzodiazepines
should be administered immediately, while investigation of potential intrathecal pump malfunction
is performed. An infectious workup can also be undertaken, but recognition of the potential lifethreatening consequence of baclofen withdrawal and immediate administration of oral baclofen
and intravenous benzodiazepines is the critical first step.
References:
Watve SV, Sivan M, Raza WA, Jamil FF. Management of acute overdose or withdrawal state in
intrathecal baclofen therapy. Spinal Cord. 2012 Feb;50(2):107-11. doi: 10.1038/sc.2011.112. Epub
2011 Oct 18. PMID: 22006082.
Pubmed Web link
https://pubmed.ncbi.nlm.nih.gov/22006082/
Saulino M, Anderson DJ, Doble J, Farid R, Gul F, Konrad P, Boster AL. Best Practices for
Intrathecal Baclofen Therapy: Troubleshooting. Neuromodulation. 2016 Aug;19(6):632-41. doi:
10.1111/ner.12467. Epub 2016 Jul 19. PMID: 27434299.
Pubmed Web link
https://pubmed.ncbi.nlm.nih.gov/27434299/

How well did you know this?
1
Not at all
2
3
4
5
Perfectly
19
Q

What is a contraindication for posterior cervical foraminotomy as a treatment for cervical
radiculopathy due to disc herniation?
Answers:
A. Kyphotic neck deformity
B. Associated osteophyte formation
C. Degenerative changes at other levels
D. Ipsilateral vocal cord palsy
E. Central disc herniation

A

Central disc herniation

Discussion:
Central disc herniation is a contraindication to posterior foraminotomy for disc herniation related
radiculopathy. There is too much risk of spinal cord retraction, and the disc should be approached
anteriorly. Associated osteophyte formation is not a problem and is addressed nicely by the
procedure. Degenerative changes at multiple levels are not atypical, and a posterior foraminotomy
should not affect their role in this patient’s neck function. Vocal cord palsy is not a contraindication
for a posterior foraminotomy (and may actually argue for a posterior approach). Kyphotic deformity
is not a contraindication for posterior foraminotomy.
References:
nul

How well did you know this?
1
Not at all
2
3
4
5
Perfectly
20
Q

A 64-year old man with history of metastatic renal cell carcinoma presents with mild back pain,
which is not affected by movement. MRI shows a T9 lytic metastasis without significant loss of
height or posterior element involvement with preservation of spinal alignment. There is moderate
epidural extension of the tumor with posterior displacement of the spinal cord, without
compression. In general, the next most appropriate step in management for this lesion is:
Answers:
A. Systemic chemotherapy alone
B. Palliative external beam radiotherapy
C. Stereotactic body radiotherapy alone
D. Observation
E. Surgical decompression with postoperative adjuvant radiotherapy

A

Surgical decompression with postoperative adjuvant radiotherapy

Discussion:
The NOMS criteria can be applied in the assessment of optimal treatment paradigms for spinal
metastatic disease. In brief, the advent of SRS has resulted in dramatically improved local control
rates for spinal metastatic disease, even among classically radioresistant tumors, such as RCC.
Per the description, this patient’s metastatic tumor displaces the spinal cord, resulting in ESCC
Grade 3 spinal cord compression. Although SRS can deliver durable tumor control, failures are
commonly correlated with underdosing of radiation to margins. Dose constraints at the tumorspinal cord margin, therefore, prevent adequate treatment. Therefore, in order to achieve
expeditious decompression and durable tumor control, surgical decompression (so-called
separation surgery) is indicated followed by adjuvant SRS.
References:
null

How well did you know this?
1
Not at all
2
3
4
5
Perfectly
21
Q

Which of the following procedures achieves the greatest degree of correction for severe spinal
deformity?
Answers:
A. Pedicle subtraction osteotomy
B. Transforaminal lumbar interbody fusion
C. Ponte osteotomy
D. Lateral interbody fusion with release of anterior longitudinal ligament
E. Vertebral column resection

A

Vertebral column resection

Discussion:
Vertebral column resection (VCR) allows for the greatest degree of correction, generally 45-70
degrees per level. VCR also allows for correction in both the sagittal and coronal planes. A pedicle
subtraction osteotomy (PSO) is another type of 3-column osteotomy generally performed in the
lower lumbar spine which allows for sagittal plane correction of 30-40 degrees. It can be performed
in conjunction with a discectomy above the PSO (extended PSO), and it can be performed
asymmetrically for coronal plane correction. Ponte osteotomies, or posterior column osteotomies,
allow for 5-10 degrees of correction. While interbody fusions, particularly with release of the
anterior longitudinal ligament can allow for significant correction, they require a mobile spine and
still do not provide as much correction as a vertebral column resection.
References:
Schwab F, Blondel B, Chay E, Demakakos J, Lenke L, Tropiano P, Ames C, Smith JS, Shaffrey CI,
Glassman S, Farcy JP, Lafage V. The comprehensive anatomical spinal osteotomy classification.
Neurosurgery. 2015 Mar; 76 Suppl 1:S33-41.
Pubmed Web link
https://pubmed.ncbi.nlm.nih.gov/25692366/
Enercan M, Ozturk C, Kahraman S, Sarier M, Hamzaoglu A, Alanay A. Osteotomies/spinal column
resections in adult deformity. Eur Spine J. 2013. Mar;22 Suppl 2(Suppl 2):S254-64.
Pubmed Web link
https://pubmed.ncbi.nlm.nih.gov/22576156/

How well did you know this?
1
Not at all
2
3
4
5
Perfectly
22
Q

The dens most typically becomes fused to the body of the axis at which of the following ages?
Answers:
A. 20 years
B. Birth
C. 6 months
D. 2 years
E. 6-9 years

A

6-9 years

Discussion:
Development of the axis takes place beginning in utero and continuing after birth, all the way into
adolescence. At birth the dens is separated from the axis body by a neural synchondrosis, and is
present in most children less than 3-4 years. By age 6-9 years the dens fuses with the axis body in
most children, while fusion of the secondary ossification centers of the dens continues into early
adolescence.
References:
Ganey TM, Ogden JA. Development and maturation of the axial skeleton. In Weinstein SL, Ed.
The Pediatric Spine: Principles and Practice, 2nd Ed. New York: Raven; 1994:3-54. Menezes AH.
Craniocervical developmental anatomy and its implications. Child’s Nervous System. 2008
Oct;24(10):1109-22.

How well did you know this?
1
Not at all
2
3
4
5
Perfectly
23
Q

A 65-year-old woman is evaluated for radicular pain into the right upper extremity. Physical
examination shows weakness of the abduction of the right fifth finger. This presentation can be due
to involvement of which of the following nerve roots?
Answers:
A. C8
B. C5
C. C7
D. T1
E. C6

A

T1

Discussion:
A T1 root injury would cause weakness of 5th digit abduction. Injury patters for other roots: C5:
elbow flexion (biceps brachii, brachialis) C6: wrist extension (extensor carpi radialis brevis and
longus) C7: elbow extension (triceps) C8: finger flexion (flexor digitorum profundus).
References:
Bertalanffy H, Eggert HR. Complications of anterior cervical discectomy without fusion in 450
consecutive patients. Acta Neurochir (Wien). 1989;99(1-2):41-50. Ebraheim NA, Lu J, Yang H, et
al. Vulnerability of the sympathetic trunk during the anterior approach to the lower cervical spine.
Spine (Phila Pa 1976). 2000 Jul 1;25(13):1603-6.

How well did you know this?
1
Not at all
2
3
4
5
Perfectly
24
Q

Axons from the dorsal root ganglia of which of the following segments of the spinal cord ascend
the gracile tract?
Answers:
A. C5
B. T1
C. L3
D. T5
E. C2

A

L3

Discussion:
The fasciculus gracilis carries ascending sensory input from heavily myelinated sensory fibers
originating below T6, while the fasciulus cuneatus recieves input from the cervical spine and upper
6 thoracic segments.
References:
Kirazlı Ö, Solmaz B, Çavdar S. The contributions to the human dorsal column tracts from the
spinal cord laminae. J Integr Neurosci. 2016 Sep;15(3):337-345. doi:
10.1142/S0219635216500217. Epub 2016 Oct 24. PMID: 27774835. Chopra S, Tadi P.
Neuroanatomy, Nucleus Gracilis. [Updated 2020 Aug 15]. In: StatPearls [Internet]. Treasure Island
(FL): StatPearls Publishing; 2021 Jan-. Available from: https://www.ncbi.nlm.nih.gov/books
/NBK546640/. Diaz E, Morales H. Spinal Cord Anatomy and Clinical Syndromes. Semin
Ultrasound CT MR. 2016 Oct;37(5):360-71. doi: 10.1053/j.sult.2016.05.002. Epub 2016 May 6.
PMID: 27616310.

How well did you know this?
1
Not at all
2
3
4
5
Perfectly
25
Q

A 57-year-old hypertensive man is evaluated because of a nine-month history of progressive gait
instability. Examination shows long tract signs. MR imaging of the thorax and spinal angiography
show a spinal dural arteriovenous fistula (sDAVF) supplied by the left T9 segmental artery, which
also supplies the artery of Adamkiewicz. Which of the following is the most appropriate
management?
Answers:
A. Propranolol
B. Surgical occlusion of the fistula via a laminectomy
C. Stereotactic radiosurgery to the lesion
D. Endovascular occlusion of the fistula
E. Physical therapy

A

Surgical occlusion of the fistula via a laminectomy

Discussion:
Occlusion of the fistula halts the progression of myelopathy and may result in improvement in
symptoms in up to 81% of patients. While both endovascular and open surgical occlusion
techniques have been described, open surgical treatment provides occlusion rates of 95% while
that of endovascular treatment is only 46% with initial treatment. In this patient, the left T9
segmental artery feeds the spinal DAVF and also the artery of Adamkiewicz, which is a
contraindication for embolization. Stereotactic radiosurgery has not been shown to be an effective
treatment for spinal DAVFs. Spinal DAVFs are easily accessed intradurally via a standard midline
laminectomy. Physical therapy has not been shown to be helpful in preventing progression of
myelopathic symptoms in patients with spinal DAVFs. Antihypertensives are not a treatment option
for spinal DAVFs.
References:
Steinmetz MP, Chow MM, Krishnaney AA, Andrews-Hinders D, Benzel EC, Masaryk TJ, Mayberg
MR, Rasmussen PA. Outcome after the treatment of spinal dural arteriovenous fistulae: a
contemporary single-institution series and meta-analysis. Neurosurgery. 2004 Jul;55(1):77-87;
discussion 87-8.
Pubmed Web link
https://pubmed.ncbi.nlm.nih.gov/15214976/
Durnford AJ, Hempenstall J, Sadek AR, Duffill J, Mathad N, Millar J, Sparrow OC, Bulters DO.
Degree and Duration of Functional Improvement on Long-Term Follow-Up of Spinal Dural
Arteriovenous Fistulae Occluded by Endovascular and Surgical Treatment. World Neurosurg. 2017
Nov;107:488-494.
Pubmed Web link
https://pubmed.ncbi.nlm.nih.gov/28774761/

How well did you know this?
1
Not at all
2
3
4
5
Perfectly
26
Q

In the upper spine, axial rotation is greatest at which of the following levels?
Answers:
A. C4-5
B. C1-C2
C. Occiput-C1
D. C2-3
E. C7-T1

A

C1-C2

Discussion:
Approximately 50% of cervical rotation occurs at the atlanto-axial level. The dens is held to the
anterior arch of C1 via the transverse ligament, which allows the dens to rotate, but limits flexion
and extension. Minimal rotation is possible at the occipito-cervical joints, 2-5 degrees. In the
subaxial cervical spine, the mean range of motion is approximately 2-10 degrees per level.
References:
Maiman DJ, Pintar FA, Groff MW, et al, eds. Concepts and Mechanisms of Biomechanics. In: Winn
HR, ed. Youmans Neurological Surgery. 5th ed. Philadelphia, PA: Elsevier, 2004. Bogduk N,
Mercer S. Biomechanics of the cervical spine. I: Normal kinematics. Clinical biomechanics. 2000
Nov 1;15(9):633-48

How well did you know this?
1
Not at all
2
3
4
5
Perfectly
27
Q

The transverse ligament inserts into which of the following?
Answers:
A. C1 lateral mass tubercle
B. Odontoid process
C. Occipital bone
D. Posterior arch of C1
E. C3 vertebral body

A

C1 lateral mass tubercle

Discussion:
The transverse ligament holds the odontoid process against the posterior aspect of the C1 anterior
arch to allow neck rotation while resisting flexion between C1 and C2. The ligament inserts onto
the medial tubercle of the C1 lateral masses. Disruption of the transverse ligament leads to
atlanto-axial instability and is radiographically associated with an increased atlanto-dental interval
(ADI).
References:
Robin AM, Yamada Y, McLaughlin LA,et al. Stereotactic Radiosurgery: The Revolutionary Advance
in the Treatment of Spine Metastases. Neurosurgery. 2017;64(CN_suppl_1):59-65
Moussazadeh N, Lis E, Katsoulakis E, Kahn S, Svoboda M, DiStefano NM, McLaughlin L, Bilsky
MH, Yamada Y, Laufer I. Five-Year Outcomes of High-Dose Single-Fraction Spinal Stereotactic
Radiosurgery. Int J Radiat Oncol Biol Phys. 2015 Oct 1;93(2):361-7. doi:
10.1016/j.ijrobp.2015.05.035. Epub 2015 May 23. PMID: 26232858.

How well did you know this?
1
Not at all
2
3
4
5
Perfectly
28
Q

The transverse ligament inserts into which of the following?
Answers:
A. Occipital bone
B. C1 lateral mass tubercle
C. C3 vertebral body
D. Posterior arch of C1
E. Odontoid process

A

C1 lateral mass tubercle

Discussion:
The transverse ligaments holds the odontoid process to the posterior aspect of the C1 anterior
arch to allow neck rotation. Mean rotational movement is 23.3 to 38.9 degrees per side. Movement
is limited by the C1–C2 articulation, the ipsilateral transverse ligament, the contralateral alar
ligament, and the capsular ligaments. Flexion is limited by the transverse ligament, while extension
is limited by the tectorial membrane and the C1–C2 articulation. The transverse ligament inserts
onto the medial tubercle of the C1 lateral mass. Disruption of the transverse ligament leads to
atlanto-axial instability.
References:
German JW, Ghanayem AJ, Benzel EC. Alexander JT. The cervical spine and cervicothoracic
junction. In: Benzel EC, ed. Spine Surgery: Techniques, Complication Avoidance and
Management. 2nd ed. Churchill-Livingstone; 2004: 278. Fielding JW, Hawkins J, Ratzan SA.
Management of atlanto-axial instability. Bull N Y Acad Med. 1976;52(7):752-760. Martin MD,
Bruner HJ, Maiman DJ. Anatomic and biomechanical considerations of the craniovertebral
junction. Neurosurgery. 2010 Mar;66(3 Suppl):2-6. doi: 10.1227/01.NEU.0000365830.10052.87.
PMID: 20173523.

How well did you know this?
1
Not at all
2
3
4
5
Perfectly
29
Q

Which of the following is the principle mechanism of action of parathyroid hormone when used in
the treatment of patients with osteoporosis?
Answers:
A. Inhibition of RANKL-RANK binding, thus preventing osteoclast inactivation
B. Activation of osteoblast function via activation of PTH/PTHrP type 1 receptors
C. Increase of trabecular bone mass via activation of estrogen receptors
D. Prevention of bone loss by maintenance of calcium levels
E. Cytotoxic injury to and interference with osteoclastic bone resorption

A

Activation of osteoblast function via activation of PTH/PTHrP type 1
receptors

Discussion:
Teriparatide, or recombinant parathyroid hormone (PTH), is the only FDA-approved anabolic agent
for the treatment of osteoporosis and works by directly stimulating osteoblast activity via the
PTH/PTHrP type 1 receptor. The other broad classes of medications used for management of
osteoporosis are either nutritional supplements (such as calcium and vitamin D) that increase
serum calcium levels or anti-resorptive (bisphosphonates, RANKL antibodies, calcitonin, or
SERMs) agents that prevent osteoclastic resorption. Bisphosphonates primarily result in cytotoxic
or metabolic injury to mature osteoclasts.
References:
Barzilai O, Fisher CG, Bilsky MH. State of the Art Treatment of Spinal Metastatic Disease.
Neurosurgery. 2018 Jun 1;82(6):757-769. doi: 10.1093/neuros/nyx567. PMID: 29481645.
Gerszten PC, Burton SA, Ozhasoglu C, Vogel WJ, Welch WC, Baar J, Friedland DM. Stereotactic
radiosurgery for spinal metastases from renal cell carcinoma. J Neurosurg Spine. 2005
Oct;3(4):288-95. doi: 10.3171/spi.2005.3.4.0288. PMID: 16266070

How well did you know this?
1
Not at all
2
3
4
5
Perfectly
30
Q

Axons from the dorsal root ganglia of which of the following segments of the spinal cord ascend
the gracile tract?
Answers:
A. C5
B. T5
C. T1
D. C2
E. L3

A

L3

Discussion:
The fasciculus gracilis carries ascending sensory input from heavily myelinated sensory fibers
originating below T6, while the fasciulus cuneatus recieves input from the cervical spine and upper
6 thoracic segments.
References:
Kirazlı Ö, Solmaz B, Çavdar S. The contributions to the human dorsal column tracts from the
spinal cord laminae. J Integr Neurosci. 2016 Sep;15(3):337-345. doi:
10.1142/S0219635216500217. Epub 2016 Oct 24. PMID: 27774835. Chopra S, Tadi P.
Neuroanatomy, Nucleus Gracilis. [Updated 2020 Aug 15]. In: StatPearls [Internet]. Treasure Island
(FL): StatPearls Publishing; 2021 Jan-. Available from: https://www.ncbi.nlm.nih.gov/books
/NBK546640/. Diaz E, Morales H. Spinal Cord Anatomy and Clinical Syndromes. Semin
Ultrasound CT MR. 2016 Oct;37(5):360-71. doi: 10.1053/j.sult.2016.05.002. Epub 2016 May 6.
PMID: 27616310.

How well did you know this?
1
Not at all
2
3
4
5
Perfectly
31
Q

Which of the following structures contains fibers that contribute to Lissauer’s tract?
Answers:
A. Sympathetic chain
B. Medial longitudinal fasciculus
C. Dorsal root ganglia
D. Spinal accessory nerve
E. Ventral horn of spinal cord

A

Dorsal root ganglia

Discussion:
Lissauer’s tract is a pathway formed by small unmyelinated and poorly myelinated fibers in
peripheral nerves that enter the lateral bundle of the dorsal root, and subsequently ascend or
descend up to several segments to eventually terminate in the ipsilateral dorsal horn.
References:
(2013) Lissauer’s Tract. In: Gebhart G.F., Schmidt R.F. (eds) Encyclopedia of Pain. Springer,
Berlin, Heidelberg. https://doi.org/10.1007/978-3-642-28753-4_201173. Peirs C, Dallel R, Todd AJ.
Recent advances in our understanding of the organization of dorsal horn neuron populations and
their contribution to cutaneous mechanical allodynia. J Neural Transm (Vienna). 2020
Apr;127(4):505-525. doi: 10.1007/s00702-020-02159-1. Epub 2020 Apr 2. Erratum in: J Neural
Transm (Vienna). 2020 Sep 22;: PMID: 32239353; PMCID: PMC7148279. William D. Willis,
The somatosensory system, with emphasis on structures important for pain,
Brain Research Reviews,
Volume 55, Issue 2,
2007,
Pages 297-313,
ISSN 0165-0173,
https://doi.org/10.1016/j.brainresrev.2007.05.010. LaMotte C. Distribution of the tract of Lissauer
and the dorsal root fibers in the primate spinal cord. J Comp Neurol. 1977 Apr 1;172(3):529-61.
doi: 10.1002/cne.901720308. PMID: 402397.

How well did you know this?
1
Not at all
2
3
4
5
Perfectly
32
Q

A 70-year-old man undergoes anterior screw fixation to treat a type II odontoid fracture. Which of
the following is the most likely postoperative complication?
Answers:
A. Aspiration pneumonia
B. Dysphagia
C. Neurological injury
D. Pseudoarthrosis
E. Vertebral artery injury

A

Dysphagia

Discussion:
In a study by Dailey, et al. (ref 1), a relatively high incidence of post-operative dysphagia is
encountered in the group of elderly patients requiring anterior odontoid screw placement. 35% of
the patients needed diet modification or a nasogastric tube after surgery.
Immobilization with a halo vest for type II odontoid fracture in patients older than 65 years-old has
been associated with a high incidence of aspiration pneumonia and respiratory arrest. In the cited
anterior odontoid screw placement series (ref 1), the risk for aspiration pneumonia was 11%, and
the nonunion rate was 19%.
References:
Dailey AT, Hart D, Finn MA, Schmidt MH, Apfelbaum RI. Anterior fixation of odontoid fractures in
an elderly population. J Neurosurg Spine. 2010 Jan;12(1):1-8. doi: 10.3171/2009.7.SPINE08589.
PMID: 20043755.
Pubmed Web link
https://pubmed.ncbi.nlm.nih.gov/20043755/
Joaquim AF, Patel AA. Surgical treatment of Type II odontoid fractures: anterior odontoid screw
fixation or posterior cervical instrumented fusion? Neurosurg Focus. 2015 Apr;38(4):E11. doi:
10.3171/2015.1.FOCUS14781. PMID: 25828487.
Pubmed Web link
https://pubmed.ncbi.nlm.nih.gov/25828487/

How well did you know this?
1
Not at all
2
3
4
5
Perfectly
33
Q

Axons from the dorsal root ganglia of which of the following segments of the spinal cord ascend
the gracile tract?
Answers:
A. L3
B. C2
C. T5
D. C5
E. T1

A

L3

Discussion:
Fasciculus gracilis carries ascending sensory input from heavily myelinated sensory fibers
originating below T6, while fasciculus cuneatus receives input from the cervical spine and upper 6
thoracic segments. These dorsal column tracts carry two-point tactile discrimination, vibration, and
proprioception. Fibers in these tracts are somatotopically organized, with cervical fibers
represented lateral, followed by thoracic, lumbar, and then sacral fibers medial.
References:
Lalan S, Khan M, Schlakman B, Penman A, Gatlin J, Herndon R. Differentiation of neuromyelitis
optica from multiple sclerosis on spinal magnetic resonance imaging. Int J MS Care.
2012;14(4):209-214. doi:10.7224/1537-2073-14.4.209
Tatekawa H, Sakamoto S, Hori M, Kaichi Y, Kunimatsu A, Akazawa K, Miyasaka T, Oba H, Okubo
T, Hasuo K, Yamada K, Taoka T, Doishita S, Shimono T, Miki Y. Imaging Differences between
Neuromyelitis Optica Spectrum Disorders and Multiple Sclerosis: A Multi-Institutional Study in
Japan. AJNR Am J Neuroradiol. 2018 Jul;39(7):1239-1247. doi: 10.3174/ajnr.A5663. Epub 2018
May 3. PMID: 29724765; PMCID: PMC7655447

How well did you know this?
1
Not at all
2
3
4
5
Perfectly
34
Q

Lumbar spondylolysis is common in athletes who play sports requiring which of the following
repetitive motions?
Answers:
A. Crouching
B. Axial loading
C. Extension and rotation
D. Flexion and extension
E. Sprinting

A

Extension and rotation

Discussion:
Repetitive extension and rotation of the spine can predispose to the development of lumbar
spondylolysis. Sports that involve repetitive flexion and extension can result in atypical lumbar
Scheuermann disease. Flexion and axial loading can result in disc herniation.
References:
Gerszten PC, Mendel E, Yamada Y. Radiotherapy and radiosurgery for metastatic spine disease:
what are the options, indications, and outcomes? Spine (Phila Pa 1976). 2009 Oct 15;34(22
Suppl):S78-92. doi: 10.1097/BRS.0b013e3181b8b6f5. PMID: 19829280.
Barzilai O, Fisher CG, Bilsky MH. State of the Art Treatment of Spinal Metastatic Disease.
Neurosurgery. 2018 Jun 1;82(6):757-769. doi: 10.1093/neuros/nyx567. PMID: 29481645

How well did you know this?
1
Not at all
2
3
4
5
Perfectly
35
Q

A 34-year-old man is stabbed in the interscapular region of the thoracic spine. Neurological
examination shows loss of strength in the right lower extremity, loss of pinprick sensation in the left
lower extremity, and loss of vibration in the right lower extremity. Which of the following is the most
likely underlying pathology?
Answers:
A. Cord Hemisection Syndrome
B. Anterior cord transection
C. Posterior column syndrome
D. Complete Cord Transection
E. Central cord syndrome

A

Cord Hemisection Syndrome

Discussion:
Spinal cord hemisection results in Brown-Sequard syndrome. Brown-Sequard Syndrome is
characterized by features of an ipsilateral motor loss and numbness to touch and vibration with
contralateral loss of pain and temperature below the lesion. It results from a hemisection of the
spinal cord. Posterior column syndrome is a rare syndrome in which the patient has preservation
of motor function but loss of proprioception and vibratory sense. They manifest a positive Romberg
sign on examination. Central cord syndrome is associated with hyperextension of the cervical
spine in a patient with cervical spondylosis. It is marked by a disproportionately greater impairment
of motor function in the upper extremities than in the lower extremities. The most pronounced loss
of function is seen in the hands. Sensory loss is variable. Anterior cord transcetion would result
incomplete paraplegia due to involvement of the motor nerves and loss of pain and temperature
sensation. Complete cord transection would result in all functional loss below the lesion.
References:
Bradley WG, Daroff RB, Marsden CD, et al, eds. Neurology in Clinical Practice: Principles of
Diagnosis and Management, Vol. 1. 4th ed. London: Butterworth-Heinemann, 2005:360. Daniel M.
Sciubba, James S. Harrop,
Chapter 174 - Management of Injuries of the Cervical Spine and Spinal Cord,
Editor(s): Alfredo Quiñones-Hinojosa,
Schmidek and Sweet Operative Neurosurgical Techniques (Sixth Edition),
W.B. Saunders,
2012,
Pages 1985-1992,
ISBN 9781416068396,
https://doi.org/10.1016/B978-1-4160-6839-6.10174-1.

How well did you know this?
1
Not at all
2
3
4
5
Perfectly
36
Q

A previously healthy 61-year-old man comes for evaluation because of progressive weakness in
his hands, tingling in his fingers, and gait instability over the past two months. Examination shows
atrophy and weakness of the hands, impaired rapid alternating movements, diminished pinprick
sensation in the fingers, diffuse hyperreflexia, and lower extremity spasticity. Which of the following
is the most likely diagnosis?
Answers:
A. Cervical spondylotic myelopathy
B. Peripheral neuropathy
C. Arteriovenous fistula
D. Transverse myelitis
E. Malingering

A

Cervical spondylotic myelopathy

Discussion:
The clinical presentation of this patient is consistent with cervical myelopathy. Symptoms of
cervical myelopathy include upper extremity weakness, numbness, or parasthesias, as well as gait
instability, spasticity, and extremity weakness. While there are multiple etiologies for cervical
myelopathy, the most likely in an adult > 55 years old is cervical spondylosis. Peripheral
neuropathy would not cause the severe spasticity. An arteriovenous fistula could cause weakness
and numbness, but usually the weakness would manifest on a side or an entire extremity rather
than the bilateral hands. Transverse myelitis is usually not progressive but rather an acute
phenomen.
References:
Fehlings MG, Skaf G.A review of the pathophysiology of cervical spondylotic myelopathy with
insights for potential novel mechanisms drawn from traumatic spinal cord injury. Spine. 1998 Dec
15; 23(24):2730-7. Toledano M, Bartleson JD. Cervical spondylotic myelopathy. Neurol Clin. 2013
Feb;31(1):287-305. doi: 10.1016/j.ncl.2012.09.003. PMID: 23186905

How well did you know this?
1
Not at all
2
3
4
5
Perfectly
37
Q

The approach to the anterior cervical spine can be performed from the right or left side. Risk of
which of the following is higher with a left-sided approach than with a right-sided approach?
Answers:
A. Recurrent laryngeal nerve injury
B. Carotid artery injury
C. Spinal cord injury
D. Dysphonia
E. Thoracic duct injury

A

Thoracic duct injury

Discussion:
Injury to the thoracic duct during anterior cervical discectomy and fusion (ACDF) is an exceedingly
rarely reported complication. In one large study of 9591 patients, only 0.02% incurred a thoracic
duct injury following ACDF. Both of these patients (2) underwent a left sided approach, and in the
literature most, but not all, case reports of thoracic duct injury indicate a left sided approach. It is
suggested that the cause for this increased incidence of thoracic duct injury on the left is that there
is greater variability in the course of the thoracic duct on the left side of the neck than the right.
Carotid injury and spinal cord injury can occur on either side. Recurrent laryngeal nerve injury and
dysphonia are thought to more likely occur with a right sided approach due to the the disparate
courses the nerves take in the lower neck secondary to differences in embryogenesis.
References:
Derakhshan A, Lubelski D, Steinmetz MP, Corriveau M, Lee S, Pace JR, Smith GA, Gokaslan Z,
Bydon M, Arnold PM, Fehlings MG. Thoracic duct injury following cervical spine surgery: a
multicenter retrospective review. Global spine journal. 2017 Apr;7(1_suppl):115S-9S. Yee TJ,
Swong K, Park P. Complications of anterior cervical spine surgery: a systematic review of the
literature. J Spine Surg. 2020 Mar;6(1):302-322. doi: 10.21037/jss.2020.01.14. PMID: 32309668;
PMCID: PMC7154369. Johnson MD, Matur AV, Asghar F, Nasser R, Cheng JS, Prestigiacomo CJ.
Right Versus Left Approach to Anterior Cervical Discectomy and Fusion: An Anatomic Versus
Historic Debate. World Neurosurg. 2020 Mar;135:135-140. doi: 10.1016/j.wneu.2019.12.052. Epub
2019 Dec 16. PMID: 31857270.

38
Q

A 65-year-old woman is evaluated because of severe and disabling neck pain nine months after
being involved in a motor vehicle collision. A CT scan of the cervical spine shows the non-union of
a type II sclerotic odontoid fracture. Which of the following is the most appropriate management?
Answers:
A. Halo management for 3 months
B. Management in a hard cervical collar for 3 months
C. Posterior C1-2 instrumented arthrodesis
D. Observation
E. Anterior odontoid screw fixation

A

Posterior C1-2 instrumented arthrodesis

Discussion:
The most appropriate management would be to perform posterior C1-2 instrumented arthrodesis.
The reported rate of chronic nonunion fractures of the dens after halo fixation in the elderly is
around 28%. A posterior C1-2 fixation would provide the best rate of fusion as compared to an
anterior odontoid screw fixation in the setting of chronic nonunion fracture of the dens. The
reported rate of fusion from a posterior C1-2 arthrodesis is approximately 85%. Contraindications
to anterior odontoid screw fixation include: disruption of the transverse ligament, concomitant
atlantoaxial joint injuries, fracture line parallel to screw trajectory, cervical kyphosis, barrel chest
habitus, obesity that would make the screw trajectory not possible. Anterior screw fixation should
not be used in patients with nonunion fractures in which fracture healing and/or fixation will be
impaired. Halo fixation in an elderly patient especially in the setting of a nonunion is associated
with a high morbidity. Observation alone in the setting of a nonunion dens fracture would risk
neurologic injury in otherwise intact patient. Management in a hard cervical collar is an option in
elderly patients who are unable to tolerate surgery but would be inappropriate in a patient in good
medical condition.
References:
Tashjian RZ, Majercik S, Biffl WL, Palumbo MA, Cioffi WG. Halo-vest immobilization increases
early morbidity and mortality in elderly odontoid fractures. J Trauma. 2006 Jan;60(1):199-203. doi:
10.1097/01.ta.0000197426.72261.17. PMID: 16456456.
Pubmed Web link
https://pubmed.ncbi.nlm.nih.gov/16456456/
Nourbakhsh A, Shi R, Vannemreddy P, Nanda A. Operative versus nonoperative management of
acute odontoid Type II fractures: a meta-analysis. J Neurosurg Spine. 2009 Dec;11(6):651-8. doi:
10.3171/2009.7.SPINE0991. PMID: 19951016.
Pubmed Web link
https://pubmed.ncbi.nlm.nih.gov/19951016/

39
Q

What characteristic imaging finding on MRI would best differentiate neuromyelitis optica from
multiple sclerosis?
Answers:
A. Multiple non-enhancing intramedullary spinal lesions spanning 1-2 segments
B. Optic nerve enhancement
C. Multiple enhancing intramedullary spinal lesions spanning 1-2 segments
D. An intramedullary spinal cord lesion spanning 3 or more segments
E. Presence of high burden of periventricular, juxtacortical and infratentorial demyelinating
lesions

A

An intramedullary spinal cord lesion spanning 3 or more segments

Discussion:
A longitudinally-extensive spinal cord lesion is most consistent with neuromyelitis optica (NMO)
and is helpful in differentiating it from multiple sclerosis. NMO, or Devic disease, once thought to
be a form of multiple sclerosis, is associated with bilateral optic neuritis, longitudinally-extensive
transverse myelitis, and antibodies to the aquaporin 4 water channel. NMO is more commonly
seen in non-Caucasian patients. 2006 guidelines for the diagnosis of NMO require optic neuritis,
myelitis, and two of the following: a longitudinally extensive spinal cord lesion on MRI (extending 3
or more segments), brain MRI not initially meeting criteria for MS, or NMO-IgG seropositivity. The
NMO antibody is 91% specific for the diagnosis. Differentiation between NMO and MS is important
clinically because of poorer prognosis with NMO and different treatment algorithms. NMO is
associated with more severe relapses, reduced likelihood of recovery, and more necrosis than
demyelination on pathology of the spinal cord. In transverse myelitis due to MS or as a clinically
isolated syndrome that may predate a diagnosis of MS, the lesions are typically 1-2 spinal cord
segments in length. Acute lesions may enhance and chronic lesions may not. Optic nerve
enhancement can be seen in either NMO or MS.
References:
Maroon JC, Kopitnik TA, Schulhof LA, Abla A, Wilberger JE. Diagnosis and microsurgical approach
to far-lateral disc herniation in the lumbar spine. J Neurosurg. 1990 Mar;72(3):378-82. doi:
10.3171/jns.1990.72.3.0378. PMID: 2303871.
Jane JA, Haworth CS, Broaddus WC, Lee JH, Malik J. A neurosurgical approach to far-lateral disc
herniation. Technical note. J Neurosurg. 1990 Jan;72(1):143-4. doi: 10.3171/jns.1990.72.1.0143.
PMID: 2294175

40
Q

Which of the following biomechanical terms describes the slope of a stress-strain curve?
Answers:
A. Ultimate strength
B. Modulus of elasticity
C. Necking
D. Yield strength
E. Strain hardening

A

Modulus of elasticity

Discussion:
With elastic materials, the linear slope of the stress-strain curve represents the modulus of
elasticity or Young’s modulus. Once the stress-strain passes the yield strength, the stress-strain
relationship becomes non-linear, demonstrating plastic deformation until eventual failure,
representing the ultimate strength of the material.
References:
HEARY, R., PARVATHREDDY, N., SAMPATH, S., AGARWAL, N.. Elastic modulus in the selection
of interbody implants. Journal of Spine Surgery, North America, 3, may. 2017. Available at: . Date
accessed: 08 Apr. 2021.
Pubmed Web link
https://www.ncbi.nlm.nih.gov/pmc/articles/PMC5506312/
Warburton A, Girdler SJ, Mikhail CM, Ahn A, Cho SK. Biomaterials in Spinal Implants: A Review.
Neurospine. 2020 Mar;17(1):101-110. doi: 10.14245/ns.1938296.148. Epub 2019 Nov 4. PMID:
31694360; PMCID: PMC7136103.
Pubmed Web link
https://pubmed.ncbi.nlm.nih.gov/31694360/

41
Q

Which of the following is most likely to supply blood to the spinal cord from C4 through C6?
Answers:
A. Artery of Adamkiewicz
B. T3 radicular artery
C. Posterior inferior cerebellar artery
D. Thyrocervical Trunk
E. External carotid artery

A

Thyrocervical Trunk

Discussion:
The blood supply of the cervical spinal cord arises from the vertebral arteries (anterior/posterior
spinal arteries), ascending cervical arteries (thyrocervical trunk), and the deep cervical arteries
(costocervical trunk).
References:
Bosmia AN, Hogan E, Loukas M, Tubbs RS, Cohen-Gadol AA. Blood supply to the human spinal
cord: part I. Anatomy and hemodynamics. Clinical Anatomy. 2015 Jan;28(1):52-64. D. Gupta,
Chapter 1 - Neuroanatomy,
Editor(s): Hemanshu Prabhakar,
Essentials of Neuroanesthesia,
Academic Press,
2017,
Pages 3-40,
ISBN 9780128052990,
https://doi.org/10.1016/B978-0-12-805299-0.00001-4. Binoy G. Chakravorty .Arterial supply of the
cervical spinal cord (with special reference to the radicular arteries)
First published: July 1971 https://doi.org/10.1002/ar.1091700308

42
Q

The great anterior radiculomedullary artery (artery of Adamkiewicz) is most likely to arise from
which of the following intercostal arteries?
Answers:
A. L2
B. T7
C. C7
D. T3
E. T10

A

T10

Discussion:
The artery of Adamkiewicz is the largest of the spinal medullary arteries and is critical for blood
supply of the thoracolumbar cord. Its origin is variable, arising between T9 and T12 (usually on the
left side) in 75% of the population. It has a characteristic “hairpin” curve as it travels to join the
anterior spinal artery. Injury to the artery of Adamkiewicz can result in anterior spinal syndrome.
References:
Parker SL, McGirt MJ, Garcés-Ambrossi GL, Mehta VA, Sciubba DM, Witham TF, Gokaslan ZL,
Wolinksy JP. Translaminar versus pedicle screw fixation of C2: comparison of surgical morbidity
and accuracy of 313 consecutive screws. Neurosurgery. 2009 May;64(5 Suppl 2):343-8;
discussion 348-9. doi: 10.1227/01.NEU.0000338955.36649.4F. PMID: 19404112.
Dorward IG, Wright NM. Seven years of experience with C2 translaminar screw fixation: clinical
series and review of the literature. Neurosurgery. 2011 Jun;68(6):1491-9; discussion 1499. doi:
10.1227/NEU.0b013e318212a4d7. PMID: 21346648.

43
Q

What is the enzyme deficiency in Tay-Sachs disease?
Answers:
A. Glucocerebrosidase
B. Beta-galactosidase A
C. Arylsulfatase A
D. Hexosaminidase A
E. Sphingomyelinase

A

Hexosaminidase A

Discussion:
The deficient protein in Tay-Sachs disease is hexosaminidase a. The other four answer choices
are implicated in other sphingolipidoses. Beta-galactosidase, sphingomyelinase, and arylsulfatase
are implicated in GM1-gangliosidosis, Niemann-Pick disease, and metachromatic leukodystrophy,
respectively. Glucocerebrosidase is implicated in Gaucher disease.
References:
Saunders WB. Aids to the Examination of the Peripheral Nervous System. 2000. London, UK.
Bishop KN, Varacallo M. Anatomy, Shoulder and Upper Limb, Dorsal Scapular Nerve. 2020 Aug
10. In: StatPearls [Internet]. Treasure Island (FL): StatPearls Publishing; 2021 Jan–. PMID:
29083775.

44
Q

A 61-year-old woman with terminal bronchial carcinoma is evaluated for unrelenting right chest
and shoulder pain. She has multiple metastatic lesions in the ribs, humerus, and scapula. Medical
management of her constant pain is ineffective. Which of the following is the correct spinal cord
location for percutaneous cervical cordotomy?
Answers:
A. Contralateral spinothalamic tract
B. Ispilateral spinothalamic tract
C. Dorsal column
D. Dorsal root entry zone
E. Ipsilateral corticospinal tract

A

Contralateral spinothalamic tract

Discussion:
Percutaneous cordotomy targets the contralateral spinothalamic tract in patients with unilateral
somatic pain, primarily cancer related pain. First order spinal nerve axons enter the ipsilateral
dorsal root, before they terminate in the dorsal horn. Second order axons then travel across the
anterior white commissure and ascend via the spinothalamic tract, where they are arranged
somatotopically. This is an excellent procedure for the treatment of cancer pain located at or below
the C5 dermatomal level for patients with limited life expectancy. Cordotomy often produces
immediate pain relief and often allows significant reduction in orally administered opiates.
References:
Crul BJP, Blok LM, van Egmond J, et al. The present role of percutaneous cervical cordotomy for
the treatment of cancer pain. J Headache Pain. 2005 Feb;6(1):24-9. Viswanathan A; Bruera E.
Cordotomy for treatment of cancer-related pain: patient selection and intervention timing.
Neurosurg Focus
2013;35(3):E6
Raslan AM; Cetas JS; McCartney S; Burchiel KJ. Destructive
procedures for control of cancer pain: the case for cordotomy. J Neurosurg 2011;114(1):155-70.

45
Q

A 35-year-old male is experiencing ascending bilateral lower extremity muscle weakness. His
lower extremity nerve conduction study (NCS) is significant for increased latency and temporal
dispersion. Conduction blocks are seen. Which type of disorder is he exhibiting?
Answers:
A. Amyotrophic lateral sclerosis
B. Eaton-Lambert syndrome
C. Myasthenia gravis
D. Guillain-Barre syndrome
E. De Quervain’s syndrome

A

Guillain-Barre syndrome

Discussion:
This portrays Guillain-Barre syndrome, which is a demyelinating condition characterized by
ascending weakness. Electrodiagnostics in ALS may demonstrate fibrillations, decreased motor
unit recruitment, and reduced numbers of motor unit potentials
References:
Greenberg, M. Handbook of Neurosurgery. Seventh Edition. Thieme. 2010. P.950-951
Kunam VK, Velayudhan V, Chaudhry ZA, Bobinski M, Smoker WRK, Reede DL. Incomplete Cord
Syndromes: Clinical and Imaging Review. Radiographics. 2018 Jul-Aug;38(4):1201-1222. doi:
10.1148/rg.2018170178. PMID: 29995620

46
Q

A 50-year-old man is evaluated because of a two-week history of low back pain and sciatica on the
right side. Straight-leg raising test is positive at 30 degrees and extensor hallucis longus strength
on the right side is 4/5. Which of the following disc herniations is most likely?
Answers:
A. L3-L4 paracentral disc herniation
B. L5-S1 foraminal disc herniation
C. L5-S1 paracentral disc herniation
D. L3-4 central disc herniation
E. L4-L5 far lateral disc herniation

A

L5-S1 foraminal disc herniation

Discussion:
This patient demonstrates a right L5 radiculopathy. Of those listed, the most likely lesion to cause a
right L5 radiculopathy is a right L5-S1 foraminal disc herniation. The L5 nerve exits its respective
neuroforamen at L5/S1. The classic paracentral disc herniation at this level would have resulted in
an S1 radiculopathy. Far lateral and foraminal disc herniations are most likely to affect the exiting
nerve root at that level while central and paracentral herniations most likely affect the traversing
nerve root.
References:
La Marca F, Brumblay H. Smith-Petersen osteotomy in thoracolumbar deformity surgery.
Neurosurgery. 2008 Sep;63(3 Suppl):163-170.
Mummaneni PV, Dhall SS, Ondra SL, Mummaneni VP, Berven S. Pedicle subtraction osteotomy.
Neurosurgery. 2008 Sep;63(3 Suppl):171-176.
Schwab F et al: The comprehensive anatomical spinal osteotomy classification. Neurosurgery
74:112-120, 2014

47
Q

In placing a translaminar screw as compared to other fixation techniques at C2, one is at increased
risk of?
Answers:
A. Carotid artery injury from a ventral breach
B. Vertebral artery injury from a lateral breach
C. Durotomy or spinal cord injury from a medial breach
D. Injuring the C2 nerve root from a lateral breach
E. Injuring the C3 nerve root from a lateral breach

A

Durotomy or spinal cord injury from a medial breach

Discussion:
During the placement of a translaminar screw at C2 the spinal cord is at risk if the lamina is
breached medially. Because of the directly visualized anatomy of the region and superficial screw
trajectory, there is little chance of penetrating the vertebral artery during placement of a
translaminar screw; whereas, vertebral artery involvement is a major risk factor during pedicle
screw placement at this site. A C2 nerve root injury from laminar screw over-advancement is much
less likely because the screw trajectory would take a path ventral and lateral to the C2 neural
foramen. Furthermore, the laminar junction with the pars can be directly visualized, allowing
recognition of a superficial or lateral pars breach which would occur with screw over-advancement.
References:
nul

48
Q

Which of the following structures contains fibers that contribute to Lissauer’s tract?
Answers:
A. Dorsal root ganglia
B. Sympathetic chain
C. Ventral horn of spinal cord
D. Medial longitudinal fasciculus
E. Spinal accessory nerve

A

Dorsal root ganglia

Discussion:
Lissauer’s tract is a pathway formed by small unmyelinated and poorly myelinated fibers in
peripheral nerves that enter the lateral bundle of the dorsal root, and subsequently ascend or
descend up to several segments to eventually terminate in the ipsilateral dorsal horn.
References:
(2013) Lissauer’s Tract. In: Gebhart G.F., Schmidt R.F. (eds) Encyclopedia of Pain. Springer,
Berlin, Heidelberg. https://doi.org/10.1007/978-3-642-28753-4_201173. Peirs C, Dallel R, Todd AJ.
Recent advances in our understanding of the organization of dorsal horn neuron populations and
their contribution to cutaneous mechanical allodynia. J Neural Transm (Vienna). 2020
Apr;127(4):505-525. doi: 10.1007/s00702-020-02159-1. Epub 2020 Apr 2. Erratum in: J Neural
Transm (Vienna). 2020 Sep 22;: PMID: 32239353; PMCID: PMC7148279. William D. Willis,
The somatosensory system, with emphasis on structures important for pain,
Brain Research Reviews,
Volume 55, Issue 2,
2007,
Pages 297-313,
ISSN 0165-0173,
https://doi.org/10.1016/j.brainresrev.2007.05.010. LaMotte C. Distribution of the tract of Lissauer
and the dorsal root fibers in the primate spinal cord. J Comp Neurol. 1977 Apr 1;172(3):529-61.
doi: 10.1002/cne.901720308. PMID: 402397.

49
Q

Which of the following is the mechanism of a Chance fracture (transverse fracture)?
Answers:
A. Shear
B. Compression
C. Flexion-distraction
D. Flexion-rotation
E. Extension

A

Flexion-distraction

Discussion:
Flexion-distraction injuries, also known as seat-belt fractures or Chance fractures (type B injuries in
the AO classification), are one- or two-level hyperflexion injuries hinging at the anterior aspect of
the vertebral column, either through the soft-tissue ligaments (B1) or through the bony structures
(B2).
Compression or flexion loads produces an anterior force on the anterior vertebral body. The injury
produced is an anterior wedge fracture.
The vector for extension injury causes forced posterior movement. The majority of the injury
resulted from extension are isolated fractures of the posterior elements (facet, laminar, or spinous
process). Severe force could lead to unstable fractures.
Flexion-rotation can produces a highly unstable injury to the vertebral column. The flexion load
produces an anterior vertebral body compression fracture and the rotation disrupts the facet joints.
Shear forces can be directed in any direction along the longitudinal axis of the spine and produces
severe ligamentous disruption.
References:
Lopez AJ, Scheer JK, Smith ZA, Dahdaleh NS. Management of flexion distraction injuries to the
thoracolumbar spine. J Clin Neurosci. 2015 Dec;22(12):1853-6. doi: 10.1016/j.jocn.2015.03.062.
Epub 2015 Jul 22. PMID: 26209922.
Pubmed Web link
https://pubmed.ncbi.nlm.nih.gov/26209922/
Wood KB, Li W, Lebl DR, Ploumis A. Management of thoracolumbar spine fractures. Spine J. 2014
Jan;14(1):145-64. doi: 10.1016/j.spinee.2012.10.041. Erratum in: Spine J. 2014 Aug 1;14(8):A18.
Lebl, Darren S [corrected to Lebl, Darren R]. PMID: 24332321.
Pubmed Web link
https://pubmed.ncbi.nlm.nih.gov/24332321/

50
Q

A 52-year-old man is brought to the emergency department after being involved in a motor vehicle
collision. He has an L2 sensory level; muscle strength is 4/5 in the proximal lower extremities and
4-/5 in the distal lower extremities. X-ray films show a severe L2 fracture. Which of the following is
the most likely ASIA Impairment Scale score (modified Frankel score) in this patient?
Answers:
A. D
B. B
C. A
D. E
E. C

A

D

Discussion:
The patient here has sustained an ASIA D injury, where motor function is preserved below the
neurological level, and more than half of the key muscles below the neurological level have a
muscle strength of at least grade 3. An ASIA A injury is complete, an ASIA B injury is incomplete
where sensory function is spared without motor function except for preservation of the sacral
segments. An ASIA C injury has motor function preserved below the neurological level, and more
than half of the key muscles below the neurological level have a muscle strength less than grade
3. ASIA E is normal strength.
References:
Jackson RP, Glah JJ. Foraminal and extra-forminal lumbar disc herniation: diagnosis and
treatment Spine. 1987 Jul-Aug; 12(6):577-585.
Broom MJ. Foraminal and extra-foraminal lumbar disk herniations. Clinical Orthop Relat Res. 1993
Apr ;( 289):118-126.
Lejeune JP, Hladky JP, Cotton A. Vinchon M, Christiaens JL. Foraminal lumbar disc herniation.
Experience with 83 patients Spine. 1994 Sep 1; 19(17):1905-1908.

51
Q

Non-displaced fractures of C1, known as Jefferson’s fractures, are best treated by
Answers:
A. Translaminar screws
B. Collar or halo immobilization
C. Posterior occipito-cervical fusion
D. Anterior odontoid screw fixation
E. Posterior C1-2 instrumented fusion

A

Collar or halo immobilization

Discussion:
Nondisplaced fractures of C1 are effectively treated with immobilization, and data suggest very
high (>96%) rates of fusion and healing. Although posterior cervical or occipito-cervical fusion may
be indicated in the context of unstable C1 fractures (injury to the transverse ligament), they are not
necessary for stable C1 fractures. Odontoid screw fixation has no role in the management of
isolated C1 fracture.
References:
Jagannathan J, Sherman JH, Szabo T, Shaffrey CI, Jane JA. The posterior cervical foraminotomy
in the treatment of cervical disc/osteophyte disease: a single-surgeon experience with a minimum
of 5 years’ clinical and radiographic follow-up. J Neurosurg Spine. 2009 Apr;10(4):347-56. doi:
10.3171/2008.12.SPINE08576. PMID: 19441994.
Henderson CM, Hennessy RG, Shuey HM Jr, Shackelford EG. Posterior-lateral foraminotomy as
an exclusive operative technique for cervical radiculopathy: a review of 846 consecutively operated
cases. Neurosurgery. 1983 Nov;13(5):504-12. doi: 10.1227/00006123-198311000-00004. PMID:
6316196.

52
Q

Which of the following structures contains fibers that contribute to Lissauer’s tract?
Answers:
A. Ventral horn
B. Spinal accessory nerve
C. Sympathetic chain
D. Dorsal root ganglion
E. Medial longitudinal fasciculus

A

Dorsal root ganglion

Discussion:
Lissauer’s tract is a pathway formed by small unmyelinated and poorly myelinated fibers in
peripheral nerves that enter the lateral bundle of the dorsal root, and subsequently ascend or
descend up to several segments to eventually terminate in the ipsilateral dorsal horn. Within the
dorsal horn, Lissauer’s tract terminates in Rexed lamina II (substantia gelatinosa), III, and to a
lesser extant lamina I.
References:
Purcell L, Micheli L. Low back pain in young athletes. Sports Health. 2009;1(3):212-222.
doi:10.1177/1941738109334212
Goetzinger S, Courtney S, Yee K, Welz M, Kalani M, Neal M. Spondylolysis in Young Athletes: An
Overview Emphasizing Nonoperative Management. J Sports Med (Hindawi Publ Corp).
2020;2020:9235958. Published 2020 Jan 21. doi:10.1155/2020/9235958

53
Q

A 72-year-old man with moderate myelopathy caused by spondylotic cervical spinal cord
compression is scheduled to undergo a multilevel posterior cervical decompression and fusion.
Which of the following is the most likely complication from this procedure during his inpatient stay?
Answers:
A. Spinal fluid leak (CSF leak)
B. Spinal cord injury/worsened myelopathy
C. Pulmonary complications
D. Dysphagia
E. Hematoma formation

A

Pulmonary complications

Discussion:
Inpatient complication rates of about 13% have been reported for postoperative cervical fusion for
spondylotic myelopathy in a large multicenter retrospective database review in the United States.
Pulmonary complications (3.6%) and postoperative hemorrhages or hematomas (2.3%) were the
most common complications reported. Utilization of incentive spirometry and early mobilization out
of bed are associated with decreased risk of pulmonary complications. Older age was associated
with increased risk of medical complications and adverse outcomes, with age > 84 as the highest
risk group with a 44-fold increased risk in adverse events compared to ages 18-44. In patients
greater than 80 years of age, additional risk factors for complications include history of prior
cerebrovascular disease and history of cancer. For all age groups, complications were greater for
posterior fusion versus anterior fusion. Anterior fusions were associated with a greater incidence of
dysphagia and hoarseness. Cervical spondylosis patients who presented for elective procedures
compared to those with urgent procedures had a much lower incidence of complications.
References:
Boakye M, Patil CG, Santarelli J, Ho C, Tian W, Lad SP. Cervical spondylotic myelopathy:
complications and outcomes after spinal fusion. Neurosurgery. 2008; 62:455-462.
Pubmed Web link
https://pubmed.ncbi.nlm.nih.gov/18382324/
Tamai K, Terai H, Suzuki A, Nakamura H, Yamashita M, Eguchi Y, Imagama S, Ando K, Kobayashi
K, Matsumoto M, Ishii K, Hikata T, Seki S, Aramomi M, Ishikawa T, Kimura A, Inoue H, Inoue G,
Miyagi M, Saito W, Yamada K, Hongo M, Endo K, Suzuki H, Nakano A, Watanabe K, Ohya J,
Chikuda H, Aoki Y, Shimizu M, Futatsugi T, Mukaiyama K, Hasegawa M, Kiyasu K, Iizuka H,
Nishida K, Kakutani K, Nakajima H, Murakami H, Demura S, Kato S, Yoshioka K, Namikawa T,
Watanabe K, Nakanishi K, Nakagawa Y, Yoshimoto M, Fujiwara H, Nishida N, Sakane M,
Yamazaki M, Kaito T, Furuya T, Orita S, Ohtori S. Risk factors of cervical surgery related
complications in patients older than 80 years. Spine Surg Relat Res. 2017 Nov 27;1(4):179-184.
doi: 10.22603/ssrr.1.2017-0002. PMID: 31440631; PMCID: PMC6698567.
Pubmed Web link
https://pubmed.ncbi.nlm.nih.gov/31440631/

54
Q

A 26-year-old man is brought to the emergency department after sustaining injuries to the lower
extremities in a motor vehicle collision. Motor function is partially preserved in the left leg (more
than half of the left leg muscles have less than antigravity strength) and sensation to pain and
temperature is markedly decreased. Motor function is absent in the right leg and proprioception is
markedly decreased. This patient most likely has which of the following spinal cord syndromes?
Answers:
A. Brown-Sequard syndrome
B. Conus medullaris syndrome
C. Posterior spinal cord syndrome
D. Anterior spinal cord syndrome
E. Central spinal cord injury

A

Brown-Sequard syndrome

Discussion:
The Brown-Sequard syndrome is characterized by a physiologic hemisection of the spinal cord.
Patients present with ipsilateral paralysis, ipsilateral loss of proprioception, and contralateral pain
and temperature loss. This is because pain and temperature fibers decussate locally in the spinal
cord while motor and proprioceptive fibers decussate in the medulla.
Posterior spinal cord syndrome is characterized by injury to the posterior spinal columns, which
are responsible for the proprioceptive and vibration senses. Patients usually present with ataxia,
loss of coordination, imbalance, and decreased sensation of vibration and light touch.
Central spinal cord injury usually occurs in patients with hyperextension injury. The spinal cord is
compressed by the inbuckling of the ligament flavum at an area of pre-existing cervical spinal cord
stenosis. The clinical manifestations are characterized by weakness or paralysis of the hands and
arms with relative preservation of the lower extremity strength. The injury is localized within a
somatotopically organized corticospinal tract.
The anterior spinal cord syndrome is characterized by a physiologic anterior 2/3 section of the
spinal cord that spares the posterior columns. Patients present with motor paralysis and pain and
temperature loss but are spared proprioception and vibration sense.
Conus medullaris syndrome occurs when there is compressive damage to the spinal cord from
T12-L2. It causes lower extremity motor/sensory dysfunction, bladder/bowel dysfunction, sexual
dysfunction, and/or saddle anesthesia.
References:
Kunam VK, Velayudhan V, Chaudhry ZA, Bobinski M, Smoker WRK, Reede DL. Incomplete Cord
Syndromes: Clinical and Imaging Review. Radiographics. 2018 Jul-Aug;38(4):1201-1222. doi:
10.1148/rg.2018170178. PMID: 29995620.
Pubmed Web link
https://pubmed.ncbi.nlm.nih.gov/29995620/
Molliqaj G, Payer M, Schaller K, Tessitore E. Acute traumatic central cord syndrome: a
comprehensive review. Neurochirurgie. 2014 Feb-Apr;60(1-2):5-11. doi:
10.1016/j.neuchi.2013.12.002. Epub 2014 Mar 7. PMID: 24613283.
Pubmed Web link
https://pubmed.ncbi.nlm.nih.gov/24613283/
Radcliff KE, Kepler CK, Delasotta LA, Rihn JA, Harrop JS, Hilibrand AS, Albert TJ, Vaccaro AR.
Current management review of thoracolumbar cord syndromes. Spine J. 2011 Sep;11(9):884-92.
doi: 10.1016/j.spinee.2011.07.022. Epub 2011 Sep 1. PMID: 21889419.
Pubmed Web link
https://pubmed.ncbi.nlm.nih.gov/21889419/

55
Q

A 50-year-old patient with HIV infection is evaluated because of a one-month history of night
sweats, fevers, cough, 9-kg (20-lb) weight loss, and back pain. Neurological examination shows no
abnormalities. QuantiFERON gold test is positive. Chest x-ray shows a cavitary lesion in the upper
lobe of the lung. MR imaging of the thoracolumbar spine shows T12 spondylitis with relative
sparing of the disc space. There is no significant vertebral body collapse. There is a large
paraspinal abscess. The MOST likely pathologic process is:
Answers:
A. Spinal tuberculosis
B. Discitis - Staph epidermidis 
C. Pathologic fracture
D. Osteoporotic compression fracture
E. Discitis - Staph aureus

A

Spinal tuberculosis

Discussion:
The radiographic description is classic for Pott’s disease. Pott’s disease is a vertebral body
infection caused by tuberculosis. Pott’s disease respects the disk space and can mimic a
malignancy. Staphylococcal infection is unlikely due to the fact that discitis is centered on the disc
space and generally spares the vertebral bodies (opposite to the imaging appearance described in
this case). Pathological fracture due to metastasis or osteoporotic compression fracture would not
explain the presence of a paraspinous abscess.
References:
Khanna K, Sabharwal S. Spinal tuberculosis: a comprehensive review for the modern spine
surgeon. Spine J. 2019 Nov;19(11):1858-1870. doi: 10.1016/j.spinee.2019.05.002. Epub 2019 May
15. PMID: 31102727.
Pubmed Web link
https://pubmed.ncbi.nlm.nih.gov/31102727/
Boody BS, Tarazona DA, Vaccaro AR. Evaluation and Management of Pyogenic and Tubercular
Spine Infections. Curr Rev Musculoskelet Med. 2018 Dec;11(4):643-652. doi:
10.1007/s12178-018-9523-y. PMID: 30280287; PMCID: PMC6220010.
Pubmed Web link
https://pubmed.ncbi.nlm.nih.gov/30280287/

56
Q

Which of the following types of injuries is most likely to result from a hyperextension injury to the
cervical spine?
Answers:
A. Burst fracture
B. Jefferson fracture
C. Compression fracture
D. Bilateral jumped facets
E. Hangman’s fracture

A

Hangman’s fracture

Discussion:
Hangman’s fractures classically result from hyperextension/distraction of the upper cervical spine,
with symmetric fracture of the C2 pars. Subaxial compression fractures, burst fractures, and
Jefferson fractures (C1 burst fractures) result from axial loading, sometimes with an additional
flexion force. Bilateral jumped facets result from hyperflexion and distraction force, allowing the
inferior articular process to “jump” over the superior articular process.
References:
Aarabi B, Koltz M, Ibrahimi D. Hyperextension cervical spine injuries and traumatic central cord
syndrome. Neurosurg Focus. 2008;25(5):E9.
Pubmed Web link
https://pubmed.ncbi.nlm.nih.gov/18980483/
Dvorak MF, Fisher CG, Fehlings MG, Rampersaud YR, Oner FC, Aarabi B, Vaccaro AR. The
surgical approach to subaxial cervical spine injuries: an evidence-based algorithm based on the
SLIC classification system. Spine. 2007 Nov 1:32(23):2620-9.
Pubmed Web link
https://pubmed.ncbi.nlm.nih.gov/17978665/

57
Q

Which of the following spinopelvic parameters is fixed and cannot be altered by the patient?
Answers:
A. Pelvic incidence
B. Fractional scoliosis
C. Sacral slope
D. Pelvic tilt
E. Lumbar lordosis

A

Pelvic incidence

Discussion:
Pelvic incidence is a fixed spinopelvic parameter. It is calculated as the angle between the
perpendicular line drawn from the center of the S1 endplate and the line that joins the center of the
femoral heads to the center of the S1 endplate. It is also defined by the formula: PI = PT + SS,
(PI=pelvic incidence, PT=pelvic tilt, SS=sacral slope). Sacral slope and pelvic tilt can vary with
retroversion of the pelvis, and lumbar lordosis changes with flexion and extension maneuvers.
Fractional scoliosis is a coronal plane parameter that is variable in the absence of spinal fusion.
References:
Terran J, Schwab F, Shaffrey CI, Smith JS, Devos P, Ames CP, Fu KMG, Burton D, Hostin R,
Klineberg E, Gupta M, Deviren V, Mundis G, Hart R, Bess S, Lafage V, International Spine Study
Group. The SRS-Schwab adult spinal deformity classification: assessment and clinical correlations
based on a prospective operative and nonoperative cohort. Neurosurgery. 2013 Oct;73(4):559-68.
Pubmed Web link
https://pubmed.ncbi.nlm.nih.gov/23756751/
Bess S, Schwab F, Lafage V, Shaffrey CI, Ames CP. Classifications for adult spinal deformity and
use of the Scoliosis Research Society-Schwab Adult Spinal Deformity Classification. Neurosurg
Clin N Am. 2013 Apr;24(2):185-93.
Pubmed Web link
https://pubmed.ncbi.nlm.nih.gov/23561557

58
Q

A 34-year-old man is stabbed in the interscapular region of the thoracic spine. Neurological
examination shows loss of strength in the right lower extremity, loss of pinprick sensation in the left
lower extremity, and loss of vibration in the right lower extremity. Which of the following is the most
likely underlying pathology?
Answers:
A. Conus medullaris syndrome
B. Posterior column syndrome
C. Brown-Sequard syndrome
D. Anterior spinal artery occlusion
E. Central cord syndrome

A

Brown-Sequard syndrome

Discussion:
Spinal cord hemisection results in Brown-Sequard syndrome, characterized by loss of ipsilateral
proprioception, ipsilateral strength, and contralateral pain and temperature. Contralateral sensory
deficits typically start 2-3 segments below the lesion, due to the ascension of these fibers prior to
decussation in the anterior commissure. Brown-Sequard usually occurs due to penetrating trauma,
multiple sclerosis, or spinal cord herniation.
References:
Eckhardt, M. Pathology and Current Treatment of Neurodegenerative Sphingolipidoses. Neuromol
Med 12, 362–382 (2010). https://doi-org.revproxy.brown.edu/10.1007/s12017-010-8133-7
Fernandes Filho JA, Shapiro BE. Tay-Sachs disease. Arch Neurol. 2004 Sep;61(9):1466-8. doi:
10.1001/archneur.61.9.1466. PMID: 15364698

59
Q

Instability of a C1 burst fracture is most commonly determined by which of the following factors?
Answers:
A. Displacement of the fragments
B. Angulation of the fragments
C. Number of fragments
D. Integrity of the transverse ligament
E. Involvement of the posterior C1 arc

A

Integrity of the transverse ligament

Discussion:
The transverse ligament is of critical importance in the stability of an atlas fracture. The transverse
ligament maintains the anatomic relationship between the odontoid and the anterior arch of C1.
Avulsion at the site of bony insertion is more common than failure of the midsubstance of the
ligament. Injury at either site results in a functionally incompetent structure.
References:
Mead LB 2nd, Millhouse PW, Krystal J, Vaccaro AR. C1 fractures: a review of diagnoses,
management options, and outcomes. Curr Rev Musculoskelet Med. 2016 Sep;9(3):255-62. doi:
10.1007/s12178-016-9356-5. PMID: 27357228; PMCID: PMC4958388.
Pubmed Web link
https://pubmed.ncbi.nlm.nih.gov/27357228/
Kim MK, Shin JJ. Comparison of radiological and clinical outcomes after surgical reduction with
fixation or halo-vest immobilization for treating unstable atlas fractures. Acta Neurochir (Wien).
2019 Apr;161(4):685-693. doi: 10.1007/s00701-019-03824-5. Epub 2019 Feb 2. PMID: 30710241.
Pubmed Web link
https://pubmed.ncbi.nlm.nih.gov/30710241/

60
Q

The umbilicus corresponds to which of the following dermatomes?
Answers:
A. T12
B. T9
C. L1
D. T11
E. T10

A

T10

Discussion:
The T10 dermatome typically represents the level of the umbilicus. The T9 dermatome is slightly
above the umbilicus, ¼ of the way to the xiphoid process. The T11 dermatome is midway between
the umbilicus and the inguinal ligament, and the T12 dermatomes is approximately at the midpoint
of the inguinal ligament.
References:
Kirshblum SC, Burns SP, Biering-Sorenson F, Donovan W, Graves DE, Jha A, Johansen M, Jones
L, Krassioukov A, Mulcahey MJ, Schmidt-Read M, Waring W. International standards for
neurological classification of spinal cord injury (Revised 2011). J Spinal Cord Med. 2011 Nov;
34(6):535-546.
Pubmed Web link
https://www.ncbi.nlm.nih.gov/pmc/articles/PMC3232636/
Hadley MN, Walters BC, Grabb PA, Oyesiku NM, Przybylski GJ, Resnick DK, Ryken TC. Clinical
assessment after acute cervical spinal cord injury. Neurosurgery. 2002 Mar:50(3 Suppl):S21-9.
Pubmed Web link
https://pubmed.ncbi.nlm.nih.gov/12431283/

61
Q

A 56-year-old man comes to the office because he has had fever and severe pain in the midthoracic spine that has been worsening for the past three weeks. Physical examination shows no
abnormalities. MR scan is shown. Analysis of needle aspirate is positive for Staphylococcus
aureus. Which of the following is the most appropriate next step?
Answers:
A. Needle biopsy
B. Debridement and internal fixation
C. Antibiotic therapy
D. Debridement 
E. Tagged WBC scan

A

Antibiotic therapy

Discussion:
Generally uncomplicated lumbar discitis/osteomyelitis can be treated with 6 weeks of IV antibiotics
and bracing (for comfort). Treatment can vary depending on the sensitivity of the organism;
antibiotic therapy may exceed 6 weeks. Internal fixation is generally not indicated, nor is open
debridement in uncomplicated cases. Positive blood cultures obviate the need for needle biopsy.
Compressive epidural abscess, neurologic deficit, and/or progressive bony destruction/deformity
may necessitate surgical intervention. Barring bony destruction and deformity, bedrest is comfort
driven and the patient should be mobilized in a brace when capable. Further imaging studies, such
as a tagged WBC scan, will not change the next step in management.
References:
Tai HC, Chen WL, Huang CC, Chen JH, Wu YL. Spontaneous septic diskitis: a common complaint
with a serious pathologic cause that should not been overlooked. Am J Emerg Med. 2008
May;26(4):514.e5-6. doi: 10.1016/j.ajem.2007.08.007. PMID: 18410828.
Pubmed Web link
https://pubmed.ncbi.nlm.nih.gov/18410828/
Tyler KL. Acute pyogenic diskitis (spondylodiskitis) in adults. Rev Neurol Dis. 2008
Winter;5(1):8-13. PMID: 18418317.
Pubmed Web link
https://pubmed.ncbi.nlm.nih.gov/18418317/

62
Q

When nociceptors are activated, which of the following is the primary neurotransmitter facilitating
synaptic transmission to dorsal horn spinal cord neurons?
Answers:
A. GABA
B. Norepinephrine
C. Seratonin
D. Glutamate
E. Dopamine

A

Glutamate

Discussion:
Nociceptors are excitatory neurons that release glutamate as their primary neurotransmitter in
addition to various peptides such as somatostatin. Pain is detected by two different types of
peripheral nociceptor neurons, C-fiber nociceptors with slowly conducting unmyelinated axons,
and A-delta nociceptors with thinly myelinated axons. In times of injury and inflammation,
nociceptors become sensitized, discharge spontaneously, and produce ongoing pain. Prolonged
firing of C-fiber nociceptors causes release of glutamate which acts on N-methyl-D-aspartate
(NMDA) receptors in the spinal cord. Activation of NMDA receptors causes the spinal cord neuron
to become more responsive to all of its inputs, resulting in central sensitization. NMDA-receptor
antagonists, such as dextromethorphan, can suppress central sensitization in experimental
animals.
References:
Dubin, A. E., & Patapoutian, A. (2010). Nociceptors: the sensors of the pain pathway. The Journal
of clinical investigation, 120(11), 3760–3772. https://doi.org/10.1172/JCI42843. Bennett GJ.
Update on the neurophysiology of pain transmission and modulation: focus on the NMDA-receptor.
J Pain Symptom Manage. 2000 Jan;19(1 Suppl):S2-6. doi: 10.1016/s0885-3924(99)00120-7.
PMID: 10687331. Khan A, Khan S, Kim YS. Insight into Pain Modulation: Nociceptors Sensitization
and Therapeutic Targets. Curr Drug Targets. 2019;20(7):775-788. doi:
10.2174/1389450120666190131114244. PMID: 30706780.

63
Q

A 60-year-old postmenopausal woman comes to the clinic with questions regarding osteoporosis
and her upcoming spinal fusion procedure. Which of the following studies is the most appropriate
initial test?
Answers:
A. Flexion/extension x-rays
B. Magnetic resonance imaging
C. Computer tomography
D. Dual energy X-ray absorptiometry (DEXA)
E. EOS films

A

Dual energy X-ray absorptiometry (DEXA)

Discussion:
According to the National Osteoporosis Foundation, the major risk factors for osteoporosis (and
related fractures) in Caucasian postmenopausal women are: 1) Personal history of fracture as an
adult, 2) History of fragility fracture in a first-degree relative, 3) Low body weight (about 127 lbs), 4)
Current smoking, and 5) Use of oral corticosteroid therapy for more than 3 months. In patients with
these risk factors (and generally in patients undergoing instrumented fusion procedures), DEXA is
the screening methodology of choice to assess bone density. While bone density measurements
can be made on computed tomography scans, DEXA remains the preferred modality at this point.
References:
U.S. Preventive Services Task Force. Screening for osteoporosis: U.S. preventive services task
force recommendation statement. Ann Intern Med. 2011 Mar 1;154(5):356-364.
Pubmed Web link
https://pubmed.ncbi.nlm.nih.gov/21242341/
Wilkins C. Osteoporosis screening and risk management. Clin Interv Aging. 2007;2(3)389-94.
Pubmed Web link
https://pubmed.ncbi.nlm.nih.gov/18044189/

64
Q

Far lateral (or zone 3) disc herniation at L4-5 would most likely affect function in which muscle?
Answers:
A. Extensor hallucis
B. Quadriceps femoris
C. Gluteus maximus
D. Gastrocnemius
E. Iliopsoas

A

Quadriceps femoris

Discussion:
Far lateral disc herniation at L4-5 is most likely to affect function of the quadriceps. Far lateral or
zone 3 lumbar disc herniation will most often affect the exiting nerve root at the pathologic level. At
L4-5 this will be the L4 nerve root. The quadriceps femoris receives significant innervation from the
L4 nerve root (and L3 through the femoral nerve). Zone 3 of the lumbar nerve root describes the
root lateral to the pedicle. Zone 2 represents the nerve root in the foramen or below the pedicle.
Zone 1 represents the region of nerve root medial to the pedicle and under the sub-articular recess
or the lateral recess. The iliopsoas is innervated by L1-3. Extensor hallucis is innervated by L5.
Gastrocnemius is innervated by S1, and gluteus maximus is innervated by L5-S2 from the sacral
plexus.
References:
null

65
Q

A patient has a wound infection after undergoing a spinal surgical procedure and is receiving
antibiotics. Which of the following test results of C-reactive protein (CRP) and erythrocyte
sedimentation rate (ESR) is most likely to indicate that the infection is resolving?
Answers:
A. ESR response is more likely than CRP response to indicate that the infection is resolving
B. Increase in CRP and increase in ESR
C. White blood cell (WBC) count is more reliable than CRP and ESR to indicate that the
infection is resolving
D. Unchanged CRP and decrease in ESR
E. Decrease in CRP and unchanged ESR

A

Decrease in CRP and unchanged ESR

Discussion:
C-reactive protein (CRP) has been shown to decrease in response to antibiotic therapy correlating
with clinical improvement in the setting of postoperative spinal wound infection. While erythrocyte
sedimentation rate (ESR) is usually elevated in postoperative spinal infections, it does not
demonstrate a reliable decrease correlative with clinical improvement and response to antibiotic
therapy. The ESR can remain elevated in the presence of a normal CRP despite clinical resolution
of postoperative wound infection. White blood cell (WBC) count is highly variable and can remain
elevated despite clinical improvement and response to antibiotic therapy, or it can remain in
normative range despite clear infection.
References:
Khan MH, Smith PN, Rao N, Donaldson WF. Serum C-reactive protein levels correlate with clinical
response in patients treated with antibiotics for wound infections after spinal surgery. Spine J. 2006
May-Jun;6(3):311-5. doi: 10.1016/j.spinee.2005.07.006. PMID: 16651226.
Pubmed Web link
https://pubmed.ncbi.nlm.nih.gov/16651226/
Mok JM, Pekmezci M, Piper SL, Boyd E, Berven SH, Burch S, Deviren V, Tay B, Hu SS. Use of
C-reactive protein after spinal surgery: comparison with erythrocyte sedimentation rate as predictor
of early postoperative infectious complications. Spine (Phila Pa 1976). 2008 Feb 15;33(4):415-21.
doi: 10.1097/BRS.0b013e318163f9ee. PMID: 18277874.
Pubmed Web link
https://pubmed.ncbi.nlm.nih.gov/18277874/

66
Q

A 46-year-old woman with Meyerding Grade 3 L5-S1 isthmic spondylolisthesis undergoes an L5-
S1 transforaminal lumbar interbody fusion. Immediately following the use of instrumentation to
achieve a complete reduction, there is strong spontaneous EMG activity arising bilaterally from the
L5 nerve roots. Which of the following is the most appropriate course of action?
Answers:
A. Reverse the reduction maneuver and end surgery immediately.
B. Reverse the reduction maneuver and explore the dorsal and ventral L5 nerve root surface
for compression source
C. Take an xray of the reduction maneuver
D. Place rods to hold the reduction and proceed with surgery
E. Utilize a handheld direct probe stimulator to assess the function of the L5 nerve roots

A

Reverse the reduction maneuver and explore the dorsal and ventral L5
nerve root surface for compression source

Discussion:
In high-grade isthmic spondylolisthesis at the lumbosacral junction, the L5 nerve roots are
compressed dorsally by the fibrinous soft tissue of the pars defect and ventrally by either disc
herniation and/or the sacrum. Reduction of the spondylolisthesis without first decompressing the
L5 nerve roots fully will lead to elevated risk of intraoperative neuromonitoring alerts such as
spontaneous EMG activity. A maneuver which results in spontaneous EMG activity is highly
predictive of neurologic injury if left untreated. Therefore, the initial step should be to reverse the
most recent maneuver, in this case reverse the reduction, and explore the course of the L5 nerve
root to evaluate for any compression sources. Notably, far-lateral compression of the L5 nerve root
outside of the neuroforamina by the sacral ala is common in high-grade L5-S1 spondylolisthesis
and should be assessed as a source of nerve root compression. Performing a Gill laminectomy
followed by completing a full foraminotomy will help relieve any dorsal source of L5 nerve root
compression prior to attempting spondylolisthesis reduction. If all decompression has been
completed and no residual compression sources are found, but neuromonitoring alerts persist if
reduction is attempted, then the reduction should be of lesser magnitude until neuromonitoring
indicates stability. Transforaminal lumbar interbody fusion (TLIF) approach is associated with
decreased operative time, blood loss, and lower complication rate compared to posterior lumbar
interbody fusion (PLIF) approach in the treatment of lumbar spondylolisthesis.
References:
Rengachary SS, Balabhandra R. Reduction of spondylolisthesis. Neurosurg Focus. 2002 Jul
15;13(1):E2. doi: 10.3171/foc.2002.13.1.3. PMID: 15916409.
Pubmed Web link
https://pubmed.ncbi.nlm.nih.gov/15916409/
de Kunder SL, van Kuijk SMJ, Rijkers K, Caelers IJMH, van Hemert WLW, de Bie RA, van
Santbrink H. Transforaminal lumbar interbody fusion (TLIF) versus posterior lumbar interbody
fusion (PLIF) in lumbar spondylolisthesis: a systematic review and meta-analysis. Spine J. 2017
Nov;17(11):1712-1721. doi: 10.1016/j.spinee.2017.06.018. Epub 2017 Jun 21. PMID: 28647584.
Pubmed Web link
https://pubmed.ncbi.nlm.nih.gov/28647584/

67
Q

The dorsal scapular nerve most commonly arises from which structure?
Answers:
A. C6 root
B. C5 root
C. C7 root
D. Upper trunk
E. Middle trunk

A

C5 root

Discussion:
The dorsal scapular nerve emerges from the posterior aspect of the C5 nerve root, just outside the
neural foramen. It courses posteriorly through the scalene muscles to innervate the rhomboid
muscles
References:
nul

68
Q

The great anterior radiculomedullary artery (artery of Adamkiewicz) is most likely to arise from
which of the following intercostal arteries?
Answers:
A. T3
B. T7
C. L2
D. T10
E. C7

A

T10

Discussion:
The artery of Adamkiewicz is the largest of the spinal medullary arteries and provides critical blood
supply to the thoracolumbar cord. Its origin is variable, with 75% of the population having it arise
between T9 and T12. Anterior cord syndrome (also called anterior spinal artery syndrome) most
commonly occurs due to an interrupted supply of the anterior spinal artery or the Artery of
Adamkiewicz (its major supplier), which has a less efficient supply compared to the 2
posterolateral spinal arteries. This syndrome affects the anterior two-thirds of the spinal cord,
which includes the majority of the anterior and lateral white matter funiculi, the central gray matter,
the bilateral lateral and anterior horns, and the bases of the posterior horns. This results in bilateral
loss of motor function (flaccid paralysis at level of lesion and spastic paralysis below the lesion)
from loss of the corticospinal tract and anterior horn, bilateral loss of pain and temperature one
level below the lesion from loss of the spinothalamic tract, and sexual dysfunction and urinary and
fecal incontinence from loss of descending autonomic tracts. Sensations of touch, vibration, and
proprioception remain intact since the posterior white matter columns are spared.
References:
Bosmia AN, Hogan E, Loukas M, Tubbs RS, Cohen-Gadol AA. Blood supply to the human spinal
cord: part I. Anatomy and hemodynamics. Clinical Anatomy. 2015 Jan;28(1):52-64. Murthy NS,
Maus TP, Behrns CL. Intraforaminal location of the great anterior radiculomedullary artery (artery
of Adamkiewicz): a retrospective review. Pain Med. 2010;11:1756-64.
Charles YP, Barbe B, Beaujeux R, Boujan F, Steib JP. Relevance of the anatomical location of the
Adamkiewicz artery in spine surgery. Surg Radiol Anat. 2011;33:3-9.

69
Q

A 56-year-old man presents with a right L5 radiculopathy of eight weeks duration despite
aggressive medical management with nonsteroidal anti-inflammatory drugs, physical therapy, and
a right extraforaminal L5 nerve block. Physical examination shows a right straight-leg raise at 60
degrees and 3+/5 weakness of the right extensor hallucis longus and anterior tibialis. BMI is
55 kg/m2. Lumbar MR imaging shows a large right disc herniation at L4-5. Alignment and the
facets at L4-5 are normal. Which of the following is the most appropriate intervention?
Answers:
A. L4-5 TLIF
B. L4-5 XLIF
C. L4 laminectomy
D. Tubular L4-5 microdiscectomy
E. L4-5 open microdiscectomy

A

Tubular L4-5 microdiscectomy

Discussion:
In patients with a lumbar disc herniation, normal sagittal alignment and normal facet anatomy,
surgical treatment is primarily with a microdiscectomy. Fusion is generally not indicated in these
patients. Obese patients with lumbar disc herniations present a particular challenge due to depth
of tissue needed to traverse in order to access the disc fragment. With tubular microsurgery, obese
patients experienced the same or equally beneficial outcome, compared to nonobese patients,
while incision lengths, blood loss, operative times, and length of stay were less when compared to
open procedures. Thus tubular microdiscectomy offers advantages versus open microdiscectomy
in obese patients as with decreased operative times, blood loss and incision length, there are
decreased rate of infection and hematoma. A laminectomy alone may not fully decompress the
nerve depending on the relationship of the the disc hernation to the facet and foramen.
References:
Park P, Upadhyaya C, Garton HJ, et al. The impact of minimally invasive spine surgery on
perioperative complications in overweight or obese patients. Neurosurgery. 2008 Mar;62(3):693-9;
discussion 693-9. Tomasino A, Parikh K, Steinberger J, Knopman J, Boockvar J, Härtl R. Tubular
microsurgery for lumbar discectomies and laminectomies in obese patients: operative results and
outcome. Spine. 2009 Aug 15;34(18):E664-72

70
Q

A 58-year-old woman undergoes an L4 laminectomy with an L4-5 interbody fusion and
instrumentation. The operation is prolonged by blood loss, and postoperatively she wakes up with
weakness in knee extension and foot dorsiflexion. She also has numbness over the anterior thigh
and medial lower leg. Which of the following nerves was most likely injured during the procedure?
Answers:
A. Sciatic nerve
B. L5 nerve root
C. Femoral nerve
D. Common peroneal nerve
E. L4 nerve root

A

L4 nerve root

Discussion:
The only nerve that could be responsible for all of these combined deficits is the L4 nerve root. The
L4 nerve root contributes to both the quadriceps muscle for knee extension via the femoral nerve
and the tibialis anterior muscle for foot dorsiflexion via the deep peroneal nerve, a terminal branch
of the common peroneal nerve which arises from the sciatic nerve. The anterior thigh and medial
lower leg are part of the L4 dermatome, receiving innervation from sensory branches of the
femoral nerve including the medial and intermediate cutaneous nerves of the thigh and the
saphenous nerve.
References:
Greenberg MS, ed. Handbook of Neurosurgery. 5th ed. Thieme Medical Publishers, 2000:873.
Pubmed Web link
Click or tap here to enter text.
Azuelos A, Corò L, Alexandre A. Femoral nerve entrapment. Acta Neurochir Suppl. 2005;92:61-2.
doi: 10.1007/3-211-27458-8_13. PMID: 15830969.
Pubmed Web link
https://pubmed.ncbi.nlm.nih.gov/15830969/

71
Q

When nociceptors are activated, which of the following is the primary neurotransmitter facilitating
synaptic transmission to dorsal horn spinal cord neurons?
Answers:
A. Dopamine
B. Glutamate
C. Seratonin
D. GABA
E. Norepinephrine

A

Glutamate

Discussion:
Nociceptors are excitatory neurons that release glutamate as their primary neurotransmitter in
addition to various peptides such as somatostatin. Pain is detected by two different types of
peripheral nociceptor neurons, C-fiber nociceptors with slowly conducting unmyelinated axons,
and A-delta nociceptors with thinly myelinated axons. In times of injury and inflammation,
nociceptors become sensitized, discharge spontaneously, and produce ongoing pain. Prolonged
firing of C-fiber nociceptors causes release of glutamate which acts on N-methyl-D-aspartate
(NMDA) receptors in the spinal cord. Activation of NMDA receptors causes the spinal cord neuron
to become more responsive to all of its inputs, resulting in central sensitization. NMDA-receptor
antagonists, such as dextromethorphan, can suppress central sensitization in experimental
animals.
References:
Dubin, A. E., & Patapoutian, A. (2010). Nociceptors: the sensors of the pain pathway. The Journal
of clinical investigation, 120(11), 3760–3772. https://doi.org/10.1172/JCI42843. Bennett GJ.
Update on the neurophysiology of pain transmission and modulation: focus on the NMDA-receptor.
J Pain Symptom Manage. 2000 Jan;19(1 Suppl):S2-6. doi: 10.1016/s0885-3924(99)00120-7.
PMID: 10687331. Khan A, Khan S, Kim YS. Insight into Pain Modulation: Nociceptors Sensitization
and Therapeutic Targets. Curr Drug Targets. 2019;20(7):775-788. doi:
10.2174/1389450120666190131114244. PMID: 30706780

72
Q

Which of the following radiographic findings is a contraindication to cervical laminoplasty in the
treatment of cervical spondylotic myelopathy?
Answers:
A. Severe cervical stenosis
B. Ossification of the posterior longitudinal ligament
C. Degenerative disc disease
D. Osteoporosis
E. Cervical kyphosis

A

Cervical kyphosis

Discussion:
Cervical laminoplasty is a surgical option for patients with multilevel cervical stenosis and lordotic
sagittal alignment to decompress the spinal canal while leaving the protective dorsal elements in
place. It is not intended for the treatment of primary axial neck pain, as no fusion is involved.
cervical canal while leaving the protective dorsal elements in place. Although it is advocated to
maintain spinal alignment and cervical range of motion, previous studies have shown that in
individuals with kyphosis or loss of lordosis, laminoplasty can lead to progression of kyphosis and
development of neck pain.
References:
Steinmetz MP, Resnick DK. Cervical laminoplasty. Spine J. 2006 Nov-Dec;6(6 Suppl):274S-281S.
Liu G, Buchowski JM, Bunmaprasert T, et al. Revision surgery following cervical laminoplasty:
etiology and treatment strategies. Spine (Phila Pa 1976). 2009 Dec 1;34(25):2760-2768

73
Q

The left anterolateral spinothalamic tract at the T6 level transmits which of the following types of
information?
Answers:
A. Light touch
B. Stereognosis
C. Vibration
D. Proprioception
E. Pain

A

Pain

Discussion:
The lateral spinothalamic tract carries information about pain and temperature. The anterior
spinothalamic tract carries sensory information regarding crude touch. This information is carried in
slow-conducting fibres (Aδ and C fibres) in contrast to the rapidly conducting fibres carrying
information about pain and temperature. After joining the spinal cord, the fibres cross after
ascending 1–2 segments and synapse in Lissauer’s tract. From there, the fibres ascend as the
lateral or anterior spinothalamic tract, and terminate in the ventral posterior nucleus of the
thalamus. Fibres are also given off to the reticular formation and periaqueductal grey matter. The
sensory cerebral cortex receives the final projections as described above. The gracile and cuneate
tracts carry information about proprioception and light touch and stereognosis.
References:
Gulgun Kayalioglu, Chapter 10 - Projections from the Spinal Cord to the Brain, Editor(s): Charles
Watson, George Paxinos, Gulgun Kayalioglu, The Spinal Cord, Academic Press, 2009, Pages
148-167. Stewart Hendry, Steven Hsiao,
Chapter 24 - The Somatosensory System,
Editor(s): Larry R. Squire, Darwin Berg, Floyd E. Bloom, Sascha du Lac, Anirvan Ghosh, Nicholas
C. Spitzer,
Fundamental Neuroscience (Fourth Edition),
Academic Press,
2013,
Pages 531-551,
ISBN 9780123858702. Lisa Harvey,
Chapter 1 - Background information,
Editor(s): Lisa Harvey,
Management of Spinal Cord Injuries,
Churchill Livingstone,
2008,
Pages 3-33,
ISBN 9780443068584,
https://doi.org/10.1016/B978-0-443-06858-4.50007-1
https://doi.org/10.1016/B978-0-12-385870-2.00024-X.

74
Q

Which of the following T-scores defines osteoporosis?
Answers:
A. -1
B. -2
C. -3
D. 0
E. 1

A

-3

Discussion:
T-score is defined as a standard deviation measurement resulting from a bone density test. The
World Health Organization (WHO) defines osteoporosis as having a T-score lower than -2.5.
Osteopenia is defined as a T-score between -1 and -2.5.
References:
Unnanuntana A, Gladnick BP, Donnelly E, Lane JM. The assessment of fracture risk. J Bone Joint
Surg Am. 2010 Mar;92(3):743-53. Kanis JA. Diagnosis of osteoporosis and assessment of fracture
risk. The Lancet. 2002 Jun 1;359(9321):1929-36

75
Q

Which of the following is most likely to supply blood to the spinal cord from C4 through C6?
Answers:
A. Posterior inferior cerebellar artery
B. T3 radicular artery
C. External carotid artery
D. Thyrocervical trunk
E. Artery of Adamkiewicz

A

Thyrocervical trunk

Discussion:
The blood supply of the cervical spinal cord arises from the vertebral arteries (anterior/posterior
spinal arteries), ascending cervical arteries (thyrocervical trunk), and the deep cervical arteries
(costocervical trunk).
References:
Bosmia AN, Hogan E, Loukas M, Tubbs RS, Cohen-Gadol AA. Blood supply to the human spinal
cord: part I. Anatomy and hemodynamics. Clinical Anatomy. 2015 Jan;28(1):52-64. D. Gupta,
Chapter 1 - Neuroanatomy,
Editor(s): Hemanshu Prabhakar,
Essentials of Neuroanesthesia,
Academic Press,
2017,
Pages 3-40,
ISBN 9780128052990,
https://doi.org/10.1016/B978-0-12-805299-0.00001-4. Binoy G. Chakravorty .Arterial supply of the
cervical spinal cord (with special reference to the radicular arteries)
First published: July 1971 https://doi.org/10.1002/ar.1091700308

76
Q

A 57-year-old man undergoes a C5 corpectomy for severe cervical myelopathy. A fibular structural
bone graft is cut, sized, and placed, and the adjacent vertebral bodies are plated to create axial
loading on the graft. By placing the graft under loading forces, remodeling and strengthening are
facilitated during healing. This is a conceptual application of which of the following principles?
Answers:
A. Three column fixation
B. Wolff’s law
C. Pseudoarthrosis
D. Hueter-Volkmann Law
E. Osteogenesis

A

Wolff’s law

Discussion:
Wolff’s law, which was developed in the 19th century, describes the concept of bone remodeling in
response to mechanical forces. Wolff’s law hypothesizes that the trabecular structure in bone is the
result of dynamic remodeling processes controlled by mechanical loads. Thus with increased
mechanical loading across a bone graft, bone density and remodeling will increase over time. Thus
in spine fusion surgery, axial compression and loading across grafts are required to facilitate boney
overgrowth and bone strengthening during healing. Osteogenesis refers to the creation of new
bone, but this does not address the remodeling of bone that occurs with axial compression across
a graft. The Hueter-Volkmann law states that increased compression acting on a growth plate
retards bone growth and, conversely, reduced compression or tension accelerates it.
References:
Wolff J, ed. The Law of Bone Remodeling. 1st ed. Berlin Heidelberg; New York NY: Springer, 1986
(translation of the German 1892 edition). Chen JH, Liu C, You L, et al. Boning up on Wolff’s Law:
Mechanical regulation of the cells that make and maintain bone. J Biomechanics. 2010 Jan
5;43(1):108-18. Mullender MG, Huiskes R. Proposal for the regulatory mechanism of Wolff’s law. J
Orthop Res. 1995 Jul;13(4):503-12. doi: 10.1002/jor.1100130405. PMID: 7674066. Smit TH.
Adolescent idiopathic scoliosis: The mechanobiology of differential growth. JOR Spine. 2020 Jul
24;3(4):e1115. doi: 10.1002/jsp2.1115. PMID: 33392452; PMCID: PMC7770204.

77
Q

The great anterior radiculomedullary artery (artery of Adamkiewicz) is most likely to arise from
which of the following intercostal arteries?
Answers:
A. C7
B. T3
C. T7
D. T10
E. L2

A

T10

Discussion:
The artery of Adamkiewicz is the largest of the spinal medullary arteries and provides critical blood
supply to the thoracolumbar cord. Its origin is variable, with 75% of the population having it arise
between T9 and T12. Anterior cord syndrome (also called anterior spinal artery syndrome) most
commonly occurs due to an interrupted supply of the anterior spinal artery or the Artery of
Adamkiewicz (its major supplier), which has a less efficient supply compared to the 2
posterolateral spinal arteries. This syndrome affects the anterior two-thirds of the spinal cord,
which includes the majority of the anterior and lateral white matter funiculi, the central gray matter,
the bilateral lateral and anterior horns, and the bases of the posterior horns. This results in bilateral
loss of motor function (flaccid paralysis at level of lesion and spastic paralysis below the lesion)
from loss of the corticospinal tract and anterior horn, bilateral loss of pain and temperature one
level below the lesion from loss of the spinothalamic tract, and sexual dysfunction and urinary and
fecal incontinence from loss of descending autonomic tracts. Sensations of touch, vibration, and
proprioception remain intact since the posterior white matter columns are spared.
References:
Bosmia AN, Hogan E, Loukas M, Tubbs RS, Cohen-Gadol AA. Blood supply to the human spinal
cord: part I. Anatomy and hemodynamics. Clinical Anatomy. 2015 Jan;28(1):52-64. Murthy NS,
Maus TP, Behrns CL. Intraforaminal location of the great anterior radiculomedullary artery (artery
of Adamkiewicz): a retrospective review. Pain Med. 2010;11:1756-64.
Charles YP, Barbe B, Beaujeux R, Boujan F, Steib JP. Relevance of the anatomical location of the
Adamkiewicz artery in spine surgery. Surg Radiol Anat. 2011;33:3-9.

78
Q

A 24-year-old woman has weakness in the right arm and leg and left abducens nerve palsy after
being involved in a high-speed motor vehicle collision. Lateral x-ray films of the cervical spine
show upper cervical prevertebral soft tissue swelling and a basion-dental interval of 16 mm. A CT
scan shows no intracranial hemorrhage. Which of the following is the most appropriate treatment?
Answers:
A. Posterior occipitocervical fusion with fixation
B. Observation
C. Halo vest
D. Traction
E. Hard collar

A

Posterior occipitocervical fusion with fixation

Discussion:
Traumatic occipitocervical dissociation (OCD) results from ligamentous injury to the craniocervical
junction and is associated with a high rate of mortality and significant neurologic morbidity. The
diagnosis is frequently missed on initial lateral cervical spinal radiographs mainly due to
inadequate visualization of radiological landmarks and low degree of suspicion. The basion-dens
interval or BDI is abnormal in the presence of a displacement between the basion and the dens of
more than 10 mm in adults or more than 12 mm in pediatric patients.
Craniocervical fixation is the treatment of choice in most cases of traumatic OCD and has been
recommended in guidelines from the American Association of Neurological Surgeons
(AANS)/Congress of Neurological Surgeons (CNS) Joint Guidelines Committee.
The use of traction in treatment of patients with OCD is associated with a 10% risk of neurological
deterioration. It is contraindicated.
External immobilization has a high failure rate. Use of nonoperative measures alone can result in
worsening in up to 50% of patients with OCD. They should be avoided as ligamentous injury is the
main mode of failure in OCD, and it is unlikely to heal over time despite prolonged external
immobilization
References:
Bono CM, Vaccaro AR, Fehlings M, Fisher C, Dvorak M, Ludwig S, Harrop J; Spine Trauma Study
Group. Measurement techniques for upper cervical spine injuries: consensus statement of the
Spine Trauma Study Group. Spine (Phila Pa 1976). 2007 Mar 1;32(5):593-600. doi:
10.1097/01.brs.0000257345.21075.a7. PMID: 17334296.
Pubmed Web link
https://pubmed.ncbi.nlm.nih.gov/17334296/
Kasliwal MK, Fontes RB, Traynelis VC. Occipitocervical dissociation-incidence, evaluation, and
treatment. Curr Rev Musculoskelet Med. 2016 Sep;9(3):247-54. doi: 10.1007/s12178-016-9347-6.
PMID: 27255101; PMCID: PMC4958379.
Pubmed Web link
https://pubmed.ncbi.nlm.nih.gov/27255101/
Walters BC, Hadley MN, Hurlbert RJ, Aarabi B, Dhall SS, Gelb DE, Harrigan MR, Rozelle CJ,
Ryken TC, Theodore N; American Association of Neurological Surgeons; Congress of Neurological
Surgeons. Guidelines for the management of acute cervical spine and spinal cord injuries: 2013
update. Neurosurgery. 2013 Aug;60(CN_suppl_1):82-91. doi:
10.1227/01.neu.0000430319.32247.7f. PMID: 23839357

79
Q

The transverse ligament inserts into which of the following?
Answers:
A. C3 vertebral body
B. Occipital bone
C. Odontoid process
D. Posterior arch of C1
E. C1 lateral mass tubercle

A

C1 lateral mass tubercle

Discussion:
The transverse ligaments holds the odontoid process to the posterior aspect of the C1 anterior
arch to allow neck rotation. Mean rotational movement is 23.3 to 38.9 degrees per side. Movement
is limited by the C1–C2 articulation, the ipsilateral transverse ligament, the contralateral alar
ligament, and the capsular ligaments. Flexion is limited by the transverse ligament, while extension
is limited by the tectorial membrane and the C1–C2 articulation. The transverse ligament inserts
onto the medial tubercle of the C1 lateral mass. Disruption of the transverse ligament leads to
atlanto-axial instability.
References:
German JW, Ghanayem AJ, Benzel EC. Alexander JT. The cervical spine and cervicothoracic
junction. In: Benzel EC, ed. Spine Surgery: Techniques, Complication Avoidance and
Management. 2nd ed. Churchill-Livingstone; 2004: 278. Fielding JW, Hawkins J, Ratzan SA.
Management of atlanto-axial instability. Bull N Y Acad Med. 1976;52(7):752-760. Martin MD,
Bruner HJ, Maiman DJ. Anatomic and biomechanical considerations of the craniovertebral
junction. Neurosurgery. 2010 Mar;66(3 Suppl):2-6. doi: 10.1227/01.NEU.0000365830.10052.87.
PMID: 20173523.

80
Q

A 65-year-old man presents with progressive neck pain, bilateral numb, clumsy hands, and a
spastic gait. Which of the following findings is least likely to occur in patients with cervical
spondylotic myelopathy:
Answers:
A. Tongue atrophy/fasciculations
B. Decreased lower extremity muscle stretch reflexes
C. Gait instability
D. Lhermitte phenomenon
E. Positive Babinski sign

A

Tongue atrophy/fasciculations

Discussion:
Cervical spondylotic myelopathy is characterized by a combination of neck pain, numb and clumsy
hands, gait disturbance, sphincter dysfunction, and impotence. Associated physical findings may
include increased lower extremity muscle stretch reflexes, positive Babinski sign, and Lhermitte
phenomenon, all of which are nonspecific signs of myelopathy. The combination of decreased
lower extremity reflexes and positive Babinski sign, although not absolute, should prompt a
consideration of vitamin B12 deficiency (combined systems disease). Also, lumbar stenosis can be
seen concordantly with other conditions, resulting in decreased peripheral reflexes. Tongue
atrophy/fasciculations can be seen in amyotrophic lateral sclerosis (ALS) and other motor neuron
diseases.
References:
Saal JA. Dillingham MF. Gamburd RS. Fanton GS. The
pseudoradicular syndrome. Lower extremity peripheral nerve entrapment
masquerading as lumbar radiculopathy. Spine. 13(8):926-30, 1988
Robblee J, Katzberg H. Distinguishing Radiculopathies from Mononeuropathies. Front Neurol.
2016;7:111. Published 2016 Jul 13. doi:10.3389/fneur.2016.00111

81
Q

Which of the following cervical vertebrae is the most common site for the entry of the vertebral
artery into the vertebral foramen?
Answers:
A. C5
B. C3
C. C6
D. C7
E. C4

A

C6

Discussion:
The vertebral artery enters the C6 transverse foramen in approximately 93% of the population.
Anatomic variants, including entry at C7 (5% of individuals), C5, and C4 can also be seen.
References:
Argenson G, Franche P, Sylla S, et al: The vertebral arteries (segments V1 and V2). Anatomical
Clinical 1980; 2:29-41. Uchino A, Saito N, Takahashi M, Okada Y, Kozawa E, Nishi N, Mizukoshi
W, Nakajima R, Watanabe Y. Variations in the origin of the vertebral artery and its level of entry into
the transverse foramen diagnosed by CT angiography. Neuroradiology. 2013 May;55(5):585-94.
doi: 10.1007/s00234-013-1142-0. Epub 2013 Jan 24. PMID: 23344682

82
Q

Which of the following cervical vertebrae is the most common site for the entry of the vertebral
artery into the vertebral foramen?
Answers:
A. C4
B. C7
C. C5
D. C3
E. C6

A

C6

Discussion:
The vertebral artery enters the C6 transverse foramen in approximately 93% of the population.
Anatomic variants, including entry at C7 (5% of individuals), C5, and C4 can also be seen. Medial
migration of the vertebral artery towards midline occurs in approximately 7% of the population. A
hypoplastic vertebral artery is found in 10% of patients.
References:
Adzick NS, Thom EA, Spong CY, Brock JW 3rd, Burrows PK, Johnson MP, Howell LJ, Farrell JA,
Dabrowiak ME, Sutton LN, Gupta N, Tulipan NB, D’Alton ME, Farmer DL; MOMS Investigators. A
randomized trial of prenatal versus postnatal repair of myelomeningocele. N Engl J Med. 2011 Mar
17;364(11):993-1004. doi: 10.1056/NEJMoa1014379. PMID: 26369371
Tulipan N, Wellons JC 3rd, Thom EA, Gupta N, Sutton LN, Burrows PK, Farmer D, Walsh W,
Johnson MP, Rand L, Tolivaisa S, D’alton ME, Adzick NS; MOMS Investigators. Prenatal surgery
for myelomeningocele and the need for cerebrospinal fluid shunt placement. J Neurosurg Pediatr.
2015 Dec;16(6):613-20. doi: 10.3171/2015.7.PEDS15336. PMID: 26369371

83
Q

An ultrasound of a 29-year-old pregnant woman reveals a lumbar myelomeningocele in the fetus.
What is the primary difference in outcome for fetal myelomeningocele repair as opposed to postnatal repair?
Answers:
A. The odds of walking without devices or braces decreases
B. The chance of maternal pregnancy complications is not affected
C. The rate of shunt placement for hydrocephalus decreases
D. The likelihood of premature birth is not affected
E. The risk of symptomatic hindbrain herniation increases

A

The rate of shunt placement for hydrocephalus decreases

Discussion:
Fetal myelomeningocele closure is associated with decreased rate of shunt placement for
hydrocephalus. Pregnant women who are carrying fetuses with myelomeningocele commonly
have questions about fetal surgery, which was shown to be efficacious by the MOMS trial
(Management of Myelomeningocele Study). This landmark study was a multi-site, randomized trial
of pre- versus post-natal spina bifida closure. The primary outcome was a composite of death OR
need for CSF shunt placement by 12 months of age. There were multiple secondary outcomes
including maternal and neonatal complications, Chiari-related findings, walking, psychomotor
development, etc. Overall, the study showed the fetal surgery is efficacious and improves a
number of measured patient outcomes on average, but at the cost of significantly increased
perinatal complications for both mother and child. The only one of these statements that is true is
that the rate of shunt placement decreased.
References:
nul

84
Q

A 34-year-old man is stabbed in the interscapular region of the thoracic spine. Neurological
examination shows loss of strength in the right lower extremity, loss of pinprick sensation in the left
lower extremity, and loss of vibration in the right lower extremity. Which of the following is the most
likely underlying pathology?
Answers:
A. Cord Hemisection Syndrome
B. Complete Cord Transection
C. Central cord syndrome
D. Anterior cord transection
E. Posterior column syndrome

A

Cord Hemisection Syndrome

Discussion:
Spinal cord hemisection results in Brown-Sequard syndrome. Brown-Sequard Syndrome is
characterized by features of an ipsilateral motor loss and numbness to touch and vibration with
contralateral loss of pain and temperature below the lesion. It results from a hemisection of the
spinal cord. Posterior column syndrome is a rare syndrome in which the patient has preservation
of motor function but loss of proprioception and vibratory sense. They manifest a positive Romberg
sign on examination. Central cord syndrome is associated with hyperextension of the cervical
spine in a patient with cervical spondylosis. It is marked by a disproportionately greater impairment
of motor function in the upper extremities than in the lower extremities. The most pronounced loss
of function is seen in the hands. Sensory loss is variable. Anterior cord transcetion would result
incomplete paraplegia due to involvement of the motor nerves and loss of pain and temperature
sensation. Complete cord transection would result in all functional loss below the lesion.
References:
Bradley WG, Daroff RB, Marsden CD, et al, eds. Neurology in Clinical Practice: Principles of
Diagnosis and Management, Vol. 1. 4th ed. London: Butterworth-Heinemann, 2005:360. Daniel M.
Sciubba, James S. Harrop,
Chapter 174 - Management of Injuries of the Cervical Spine and Spinal Cord,
Editor(s): Alfredo Quiñones-Hinojosa,
Schmidek and Sweet Operative Neurosurgical Techniques (Sixth Edition),
W.B. Saunders,
2012,
Pages 1985-1992,
ISBN 9781416068396,
https://doi.org/10.1016/B978-1-4160-6839-6.10174-1.

85
Q

A 12 year-old with Down syndrome has abnormal flexion-extension cervical spine x-rays. The child
has no significant neurologic complaints or neck pain. On exam, the child has full range of motion
and no tenderness to palpation. The flexion-extension x-rays show a 7-8 mm atlantodental interval
in flexion which reduces to 4 mm in extension. What is the most appropriate next step?
Answers:
A. Observation with no repeat imaging necessary
B. Occipito-cervical fusion with sublaminar wires
C. C1-2 wiring with halo placement and autologous fusion
D. C1-2 transarticular screw placement and autologous fusion
E. Observation with repeat flexion-extension in the future

A

Observation with repeat flexion-extension in the future

Discussion:
The correct answer is observation with repeat flexion-extension x-rays. Children with Down
syndrome have abnormalities at C1-2 with a widened atlantodental interval (ADI) of varying
degrees. Symptomatic atlantoaxial instability occurs in approximately 1-2 % of patients.
Brockmeyer, et al. discusses 1 cm subluxation as being the threshold for fusion in the
asymptomatic Down child with widened ADI. The natural history of ligamentous laxity in this region
in Down syndrome suggests that most patients will not progress to develop mechanical instability if
initial imaging studies are normal. However, in the face of several millimeters of motion on flexionextension views, further follow up images are warranted. All of the other choices could be potential
surgical options in the face of clear instability, except occipito-cervical fusion as the abnormality is
at C1-2.
References:
Fountas KN, Kapsalaki EZ, Nikolakakos LG, Smisson HF, Johnston KW, Grigorian AA, Lee GP,
Robinson JS Jr. Anterior cervical discectomy and fusion associated complications. Spine (Phila Pa
1976). 2007 Oct 1;32(21):2310-7. doi: 10.1097/BRS.0b013e318154c57e. PMID: 17906571.
Ghogawala Z, Terrin N, Dunbar MR, et al. Effect of Ventral vs Dorsal Spinal Surgery on PatientReported Physical Functioning in Patients With Cervical Spondylotic Myelopathy: A Randomized
Clinical Trial. JAMA. 2021;325(10):942–951. doi:10.1001/jama.2021.1233

86
Q

A 60-year-old man is evaluated because of a ten-week history of back pain. Serum erythrocyte
sedimentation rate and C-reactive protein levels are elevated, but his WBC count is normal. Blood
cultures are negative. Physical examination shows normal strength and sensation, and no bowel
or bladder dysfunction. MR imaging shows discitis at L3-4 without epidural abscess or canal
compromise. Which of the following is the most appropriate next step in the management of this
patient?
Answers:
A. Disc space biopsy
B. Empiric antibiotics
C. Lumbar puncture
D. Surgical debridement
E. Lumbar brace

A

Disc space biopsy

Discussion:
The next management step should be focused on identifying the infectious organism prior to the
administration of broad-spectrum antibiotics. The first step in identifying the organism is panculturing the patient including blood cultures. Image-guided percutaneous biopsy of the vertebral
body and/or disc space is helpful when cultures are negative and has a diagnostic yield between
50 and 91%. Surgical debridement is not the first line treatment in discitis without neurologic
compression. Lumbar puncture is contraindicated in the setting of spinal infection because it can
introduce organisms into the CSF or can worsen neurological function by lowering the CSF
pressure below an area of compression. The diagnostic yield from lumbar puncture is low. The
patient’s pain can sometimes be improved by applying a lumbar brace, but this does not treat the
underlying pathology.
References:
Chew FS, Kline MJ. Diagnostic yield of CT-guided percutaneous aspiration procedures in
suspected spontaneous infectious diskitis. Radiology. 2001 Jan;218(1):211-4. doi:
10.1148/radiology.218.1.r01ja06211. PMID: 11152804.
Pubmed Web link
https://pubmed.ncbi.nlm.nih.gov/11152804/
Quiñones-Hinojosa A, Jun P, Jacobs R, Rosenberg WS, Weinstein PR. General principles in the
medical and surgical management of spinal infections: a multidisciplinary approach. Neurosurg
Focus. 2004 Dec 15;17(6):E1. doi: 10.3171/foc.2004.17.6.1. PMID: 15636566.
Pubmed Web link
https://pubmed.ncbi.nlm.nih.gov/15636566/

87
Q

What is the most common complication of anterior cervical discectomy and fusion (ACDF)?
Answers:
A. Horner’s syndrome
B. Dysphagia
C. Hoarseness
D. New radiculopathy
E. Thoracic duct injury

A

Dysphagia

Discussion:
Temporary or persistent dysphagia can occur in up to 18% of post-operative ACDF patients.
Improvement may take many months. Dysphagia appears to be secondary to local response to
traction and manipulation. Other complications and syndromes are thankfully more rare although
overall morbidity for the procedure can approach 25%. Hoarseness occurs in 4.9% of cases
reported in the literature. Esophageal injury occurs in less than 1% of cases as does Horner’s
syndrome, vertebral artery injury, and thoracic duct injury. New neurologic deficits can occur in up
to 2% of cases. Recurrent laryngeal nerve injuries (resulting in hoarseness) occur in 1-2% of
cases.
References:
null

88
Q

A 65 year-old man with recent history of renal cell carcinoma status post nephrectomy presents
with severe low thoracic back pain. He undergoes stereotactic radiosurgery, 24 Gy in a single
fraction, for a T10 vertebral body metastasis. What is the most frequent complication associated
with this treatment?
Answers:
A. Esophagitis
B. Radiculitis
C. Myelitis
D. Vertebral body fracture
E. Dermatitis

A

Vertebral body fracture

Discussion:
Single fraction or hypofractionated stereotactic radiation therapy to spinal metastases has been
shown to have high local control rates with relatively low rates of morbidity. Post-treatment
vertebral body fracture is the most commonly seen complication reported in 4-40% of patients with
around 16% of patients experiencing symptomatic fractures. Radiation-induced esophagitis,
dermatitis and radiculitis are generally seen in less than 5% of cases. Radiation-induced myelitis
has been reported in less than 1% of patients treated with stereotactic spinal radiation using typical
dosing schemes. Although many patients succumb to their metastatic disease within 5 years of
treatment, the long-term complication profile among the survivors also reflects this trend
References:
null

89
Q

The left anterolateral spinothalamic tract at the T6 level transmits which of the following types of
information?
Answers:
A. Deep pressure
B. Vibration
C. Proprioception
D. Stereognosis
E. Pain

A

Pain

Discussion:
The anterolateral tract of the spinal cord carries nociception, temperature, and non-discriminative
touch information. This information is transmitted to the ventral posterolateral (VPL) thalamus.
Vibration, stereognosis, proprioception, and deep pressure are transmitted by the dorsal column
system.
References:
Joyce NC, Carter GT. Electrodiagnosis in persons with amyotrophic lateral sclerosis. PM R.
2013;5(5 Suppl):S89-S95. doi:10.1016/j.pmrj.2013.03.020
Garg N, Park SB, Vucic S, Yiannikas C, Spies J, Howells J, Huynh W, Matamala JM, Krishnan AV,
Pollard JD, Cornblath DR, Reilly MM, Kiernan MC. Differentiating lower motor neuron syndromes.
J Neurol Neurosurg Psychiatry. 2017 Jun;88(6):474-483. doi: 10.1136/jnnp-2016-313526. Epub
2016 Dec 21. PMID: 28003344; PMCID: PMC5529975.

90
Q

A 61-year-old woman with terminal bronchial carcinoma is evaluated for unrelenting right chest
and shoulder pain. She has multiple metastatic lesions in the ribs, humerus, and scapula. Medical
management of her constant pain is ineffective. Which of the following is the correct spinal cord
location for percutaneous cervical cordotomy?
Answers:
A. Dorsal root entry zone
B. Dorsal column
C. Contralateral spinothalamic tract
D. Ispilateral spinothalamic tract
E. Ipsilateral corticospinal tract

A

Contralateral spinothalamic tract

Discussion:
Percutaneous cordotomy targets the contralateral spinothalamic tract in patients with unilateral
somatic pain, primarily cancer related pain. First order spinal nerve axons enter the ipsilateral
dorsal root, before they terminate in the dorsal horn. Second order axons then travel across the
anterior white commissure and ascend via the spinothalamic tract, where they are arranged
somatotopically. This is an excellent procedure for the treatment of cancer pain located at or below
the C5 dermatomal level for patients with limited life expectancy. Cordotomy often produces
immediate pain relief and often allows significant reduction in orally administered opiates.
References:
Crul BJP, Blok LM, van Egmond J, et al. The present role of percutaneous cervical cordotomy for
the treatment of cancer pain. J Headache Pain. 2005 Feb;6(1):24-9. Viswanathan A; Bruera E.
Cordotomy for treatment of cancer-related pain: patient selection and intervention timing.
Neurosurg Focus
2013;35(3):E6
Raslan AM; Cetas JS; McCartney S; Burchiel KJ. Destructive
procedures for control of cancer pain: the case for cordotomy. J Neurosurg 2011;114(1):155-70.

91
Q

Use of which of the following pharmacologic agents reduces blood loss and transfusion
requirements during routine craniofacial surgery in infants?
Answers:
A. Dexamethasone
B. Desmopressin
C. Tranexamic acid
D. Factor VII
E. Aminocaproic acid

A

Tranexamic acid

Discussion:
Tranexamic acid (TXA) is a lysine analog that competitively inhibits the conversion of plasminogen
to plasmin, thus inhibiting the proteolytic action of plasmin on the fibrin clot, thereby inhibiting
fibrinolysis at the surgical site and promoting clot formation. In a randomized, prospective, doubleblind, placebo-controlled study, TXA was found to significantly reduce blood loss and need for
blood transfusion in infants undergoing craniosynostosis repair. Aminocaproic acid, desmopressin,
and Factor VII all affect clotting mechanisms but are not utilized in routine craniofacial surgery due
to side effects. Dexamethasone does not play a role in blood loss inhibition.
References:
Goobie SM, Meier PM, Pereira LM, McGowan FX, Prescilla RP, Scharp LA, Rogers GF, Proctor
MR, Meara JG, Soriano SG, Zurakowski D, Sethna NF. Efficacy of tranexamic acid in pediatric
craniosynostosis surgery: a double-blind, placebo-controlled trial. Anesthesiology. 2011
Apr;114(4):862-71. doi: 10.1097/ALN.0b013e318210fd8f. PMID: 21364458.
Pubmed Web link
https://pubmed.ncbi.nlm.nih.gov/21364458/
Dadure C, Sauter M, Bringuier S, Bigorre M, Raux O, Rochette A, Canaud N, Capdevila X.
Intraoperative tranexamic acid reduces blood transfusion in children undergoing craniosynostosis
surgery: a randomized double-blind study. Anesthesiology. 2011 Apr;114(4):856-61. doi:
10.1097/ALN.0b013e318210f9e3. PMID: 21358317.
Pubmed Web link
https://pubmed.ncbi.nlm.nih.gov/21358317/

92
Q

Which of the following is most likely to supply blood to the spinal cord from C4 through C6?
Answers:
A. Thyrocervical trunk
B. T3 radicular artery
C. Artery of Adamkiewicz
D. External carotid artery
E. Posterior inferior cerebellar artery

A

Thyrocervical trunk

Discussion:
The blood supply of the cervical spinal cord arises from the vertebral arteries (anterior/posterior
spinal arteries), ascending cervical arteries (thyrocervical trunk), and the deep cervical arteries
(costocervical trunk). Radicular branches from the ascending cervical artery enter neural foramina,
usually C3-4, C4-5. Deep cervical artery branches enter foramina from C5-T1.
References:
Pizzutillo PD, Herman MJ. Cervical spine issues in Down syndrome. J Pediatr Orthop. 2005 MarApr;25(2):253-9. doi: 10.1097/01.bpo.0000154227.77609.90. PMID: 15718913.
Hengartner AC, Whelan R, Maj R, Wolter-Warmerdam K, Hickey F, Hankinson TC. Evaluation of
2011 AAP cervical spine screening guidelines for children with Down Syndrome. Childs Nerv Syst.
2020 Nov;36(11):2609-2614. doi: 10.1007/s00381-020-04855-5. Epub 2020 Aug 10. PMID:
32778937